Labor and Birth Comp, Fetal Assessment During Labor, The High Risk Newborn, Childbirth at Risk/Prelabor and Intrapartum Comp, Labor Related Comp, Ch 3...

Pataasin ang iyong marka sa homework at exams ngayon gamit ang Quizwiz!

The nurse is planning care for a preterm newborn. Which nursing diagnosis has the highest priority? 1. Tissue Integrity, Impaired 2. Infection, Risk for 3. Gas Exchange, Impaired 4. Family Processes, Dysfunctional

Answer: 3 Explanation: 3. Gas Exchange, Impaired is related to immature pulmonary vasculature and inadequate surfactant production and has the highest priority.

A woman in preterm labor at 30 weeks of gestation receives two 12-mg doses of betamethasone intramuscularly. The purpose of this pharmacologic treatment is to: a. Stimulate fetal surfactant production. b. Reduce maternal and fetal tachycardia associated with ritodrine administration. c. Suppress uterine contractions. d. Maintain adequate maternal respiratory effort and ventilation during magnesium sulfate therapy.

ANS: A Antenatal glucocorticoids given as intramuscular injections to the mother accelerate fetal lung maturity. Inderal would be given to reduce the effects of ritodrine administration. Betamethasone has no effect on uterine contractions. Calcium gluconate would be given to reverse the respiratory depressive effects of magnesium sulfate therapy. PTS: 1 DIF: Cognitive Level: Comprehension REF: 450 OBJ: Nursing Process: Planning MSC: Client Needs: Physiologic Integrity

Before the physician performs an external version, the nurse should expect an order for a: a. Tocolytic drug. b. Contraction stress test (CST). c. Local anesthetic. d. Foley catheter.

ANS: A A tocolytic drug will relax the uterus before and during version, thus making manipulation easier. CST is used to determine the fetal response to stress. A local anesthetic is not used with external version. The bladder should be emptied; however, catheterization is not necessary. PTS: 1 DIF: Cognitive Level: Comprehension REF: 460 OBJ: Nursing Process: Planning MSC: Client Needs: Physiologic Integrity

The abuse of which of the following substances during pregnancy is the leading cause of cognitive impairment in the United States? a. Alcohol b. Tobacco c. Marijuana d. Heroin

ANS: A Alcohol abuse during pregnancy is recognized as one of the leading causes of cognitive impairment in the United States. PTS: 1 DIF: Cognitive Level: Knowledge REF: 670 OBJ: Nursing Process: Assessment MSC: Client Needs: Psychosocial Integrity

The nurse providing care for the laboring woman should understand that amnioinfusion is used to treat: a. Variable decelerations. b. Late decelerations. c. Fetal bradycardia. d. Fetal tachycardia.

ANS: A Amnioinfusion is used during labor either to dilute meconium-stained amniotic fluid or to supplement the amount of amniotic fluid to reduce the severity of variable decelerations caused by cord compression. Amnioinfusion has no bearing on late decelerations, fetal bradycardia, or fetal tachycardia alterations in fetal heart rate (FHR) tracings. PTS: 1 DIF: Cognitive Level: Knowledge REF: 396 OBJ: Nursing Process: Implementation MSC: Client Needs: Health Promotion and Maintenance

With regard to the process of augmentation of labor, the nurse should be aware that it: a. Is part of the active management of labor that is instituted when the labor process is unsatisfactory. b. Relies on more invasive methods when oxytocin and amniotomy have failed. c. Is a modern management term to cover up the negative connotations of forceps-assisted birth. d. Uses vacuum cups.

ANS: A Augmentation is part of the active management of labor that stimulates uterine contractions after labor has started but is not progressing satisfactorily. Augmentation uses amniotomy and oxytocin infusion, as well as some gentler, noninvasive methods. Forceps-assisted births and vacuum-assisted births are appropriately used at the end of labor and are not part of augmentation. PTS: 1 DIF: Cognitive Level: Comprehension REF: 465 OBJ: Nursing Process: Planning MSC: Client Needs: Health Promotion and Maintenance

Which patient status is an acceptable indication for serial oxytocin induction of labor? a. Past 42 weeks gestation b. Multiple fetuses c. Polyhydramnios d. History of long labors

ANS: A Continuing a pregnancy past the normal gestational period is likely to be detrimental to fetal health. Multiple fetuses overdistend the uterus and make induction of labor high risk. Polyhydramnios overdistends the uterus, again making induction of labor high risk. History of rapid labors is a reason for induction of labor because of the possibility that the baby would otherwise be born in uncontrolled circumstances. PTS: 1 DIF: Cognitive Level: Comprehension REF: 453 OBJ: Nursing Process: Assessment MSC: Client Needs: Physiologic Integrity

A normal uterine activity pattern in labor is characterized by: a. Contractions every 2 to 5 minutes. b. Contractions lasting about 2 minutes. c. Contractions about 1 minute apart. d. A contraction intensity of about 1000 mm Hg with relaxation at 50 mm Hg.

ANS: A Contractions normally occur every 2 to 5 minutes and last less than 90 seconds (intensity 800 mm Hg) with about 30 seconds in between (20 mm Hg or less). PTS: 1 DIF: Cognitive Level: Knowledge REF: 383 OBJ: Nursing Process: Assessment MSC: Client Needs: Health Promotion and Maintenance

Which deceleration of the fetal heart rate would not require the nurse to change the maternal position? a. Early decelerations b. Late decelerations c. Variable decelerations d. It is always a good idea to change the womans position.

ANS: A Early decelerations (and accelerations) generally do not need any nursing intervention. Late decelerations suggest that the nurse should change the maternal position (lateral); variable decelerations also require a maternal position change (side to side). Although changing positions throughout labor is recommended, it is not required in response to early decelerations. PTS: 1 DIF: Cognitive Level: Comprehension REF: 391 OBJ: Nursing Process: Assessment, Planning MSC: Client Needs: Physiologic Integrity

The nurse caring for the laboring woman should understand that early decelerations are caused by: a. Altered fetal cerebral blood flow. b. Umbilical cord compression. c. Uteroplacental insufficiency. d. Spontaneous rupture of membranes.

ANS: A Early decelerations are the fetuss response to fetal head compression. Variable decelerations are associated with umbilical cord compression. Late decelerations are associated with uteroplacental insufficiency. Spontaneous rupture of membranes has no bearing on the fetal heart rate unless the umbilical cord prolapses, which would result in variable or prolonged bradycardia. PTS: 1 DIF: Cognitive Level: Comprehension REF: 391 OBJ: Nursing Process: Assessment MSC: Client Needs: Physiologic Integrity

The nurse providing care for the laboring woman comprehends that accelerations with fetal movement: a. Are reassuring. b. Are caused by umbilical cord compression. c. Warrant close observation. d. Are caused by uteroplacental insufficiency.

ANS: A Episodic accelerations in the fetal heart rate (FHR) occur during fetal movement and are indications of fetal well-being. Umbilical cord compression results in variable decelerations in the FHR. Accelerations in the FHR are an indication of fetal well-being and do not warrant close observation. Uteroplacental insufficiency would result in late decelerations in the FHR. PTS: 1 DIF: Cognitive Level: Knowledge REF: 390 OBJ: Nursing Process: Assessment MSC: Client Needs: Physiologic Integrity

An infant with severe meconium aspiration syndrome (MAS) is not responding to conventional treatment. Which highly technical method of treatment may be necessary for an infant who does not respond to conventional treatment? a. Extracorporeal membrane oxygenation b. Respiratory support with a ventilator c. Insertion of a laryngoscope and suctioning of the trachea d. Insertion of an endotracheal tube

ANS: A Extracorporeal membrane oxygenation is a highly technical method that oxygenates the blood while bypassing the lungs, thus allowing the infants lungs to rest and recover. The infant is likely to have been first connected to a ventilator. Laryngoscope insertion and tracheal suctioning are performed after birth before the infant takes the first breath. An endotracheal tube will be in place to facilitate deep tracheal suctioning and ventilation. PTS: 1 DIF: Cognitive Level: Knowledge REF: 693 OBJ: Nursing Process: Evaluation MSC: Client Needs: Physiologic Integrity

Fetal well-being during labor is assessed by: a. The response of the fetal heart rate (FHR) to uterine contractions (UCs). b. Maternal pain control. c. Accelerations in the FHR. d. An FHR above 110 beats/min.

ANS: A Fetal well-being during labor can be measured by the response of the FHR to UCs. In general, reassuring FHR patterns are characterized by an FHR baseline in the range of 110 to 160 beats/min with no periodic changes, a moderate baseline variability, and accelerations with fetal movement. Maternal pain control is not the measure used to determine fetal well-being in labor. Although FHR accelerations are a reassuring pattern, they are only one component of the criteria by which fetal well-being is assessed. Although an FHR above 110 beats/min may be reassuring, it is only one component of the criteria by which fetal well-being is assessed. More information would be needed to determine fetal well-being. PTS: 1 DIF: Cognitive Level: Comprehension REF: 400 OBJ: Nursing Process: Assessment MSC: Client Needs: Physiologic Integrity

Which infant would be more likely to have Rh incompatibility? a. Infant of an Rh-negative mother and a father who is Rh positive and homozygous for the Rh factor b. Infant who is Rh negative and whose mother is Rh negative c. Infant of an Rh-negative mother and a father who is Rh positive and heterozygous for the Rh factor d. Infant who is Rh positive and whose mother is Rh positive

ANS: A If the mother is Rh negative and the father is Rh positive and homozygous for the Rh factor, all the children will be Rh positive. Only Rh-positive children of an Rh-negative mother are at risk for Rh incompatibility. If the mother is Rh negative and the father is Rh positive and heterozygous for the factor, there is a 50% chance that each infant born of the union will be Rh positive and a 50% chance that each will be born Rh negative. PTS: 1 DIF: Cognitive Level: Comprehension REF: 679 OBJ: Nursing Process: Planning MSC: Client Needs: Health Promotion and Maintenance

With regard to injuries to the infants plexus during labor and birth, nurses should be aware that: a. If the nerves are stretched with no avulsion, they should recover completely in 3 to 6 months. b. Erb palsy is damage to the lower plexus. c. Parents of children with brachial palsy are taught to pick up the child from under the axillae. d. Breastfeeding is not recommended for infants with facial nerve paralysis until the condition resolves.

ANS: A If the nerves are stretched with no avulsion, they should recover completely in 3 to 6 months. However, if the ganglia are disconnected completely from the spinal cord, the damage is permanent. Erb palsy is damage to the upper plexus and is less serious than brachial palsy. Parents of children with brachial palsy are taught to avoid picking up the child under the axillae or by pulling on the arms. Breastfeeding is not contraindicated, but both the mother and infant will need help from the nurse at the start. PTS: 1 DIF: Cognitive Level: Comprehension REF: 665 OBJ: Nursing Process: Planning MSC: Client Needs: Physiologic Integrity

Which maternal condition is considered a contraindication for the application of internal monitoring devices? a. Unruptured membranes b. Cervix dilated to 4 cm c. External monitors in current use d. Fetus with a known heart defect

ANS: A In order to apply internal monitoring devices, the membranes must be ruptured. Cervical dilation of 4 cm permits the insertion of fetal scalp electrodes and intrauterine catheter. The external monitor can be discontinued after the internal ones are applied. A compromised fetus should be monitored with the most accurate monitoring devices. PTS: 1 DIF: Cognitive Level: Comprehension REF: 386 OBJ: Nursing Process: Planning MSC: Client Needs: Physiologic Integrity

While evaluating an external monitor tracing of a woman in active labor, the nurse notes that the fetal heart rate (FHR) for five sequential contractions begins to decelerate late in the contraction, with the nadir of the decelerations occurring after the peak of the contraction. The nurses first priority is to: a. Change the womans position. b. Notify the care provider. c. Assist with amnioinfusion. d. Insert a scalp electrode.

ANS: A Late decelerations may be caused by maternal supine hypotension syndrome. They usually are corrected when the woman turns on her side to displace the weight of the gravid uterus from the vena cava. If the fetus does not respond to primary nursing interventions for late decelerations, the nurse would continue with subsequent intrauterine resuscitation measures, including notifying the care provider. An amnioinfusion may be used to relieve pressure on an umbilical cord that has not prolapsed. The FHR pattern associated with this situation most likely reveals variable deceleration. A fetal scalp electrode would provide accurate data for evaluating the well-being of the fetus; however, this is not a nursing intervention that would alleviate late decelerations, nor is it the nurses first priority. PTS: 1 DIF: Cognitive Level: Application REF: 392 OBJ: Nursing Process: Implementation MSC: Client Needs: Physiologic Integrity

An infant at 26 weeks of gestation arrives intubated from the delivery room. The nurse weighs the infant, places him under the radiant warmer, and attaches him to the ventilator at the prescribed settings. A pulse oximeter and cardiorespiratory monitor are placed. The pulse oximeter is recording oxygen saturations of 80%. The prescribed saturations are 92%. The nurses most appropriate action would be to: a. Listen to breath sounds and ensure the patency of the endotracheal tube, increase oxygen, and notify a physician. b. Continue to observe and make no changes until the saturations are 75%. c. Continue with the admission process to ensure that a thorough assessment is completed. d. Notify the parents that their infant is not doing well.

ANS: A Listening to breath sounds and ensuring the patency of the endotracheal tube, increasing oxygen, and notifying a physician are appropriate nursing interventions to assist in optimal oxygen saturation of the infant. Oxygenation of the infant is crucial. O2 saturation should be maintained above 92%. Oxygenation status of the infant is crucial. The nurse should delay other tasks to stabilize the infant. Notifying the parents that the infant is not doing well is not an appropriate action. Further assessment and intervention are warranted before determination of fetal status. PTS: 1 DIF: Cognitive Level: Application REF: 688 OBJ: Nursing Process: Implementation MSC: Client Needs: Physiologic Integrity

The nurse caring for a laboring woman is aware that maternal cardiac output can be increased by: a. Change in position. b. Oxytocin administration. c. Regional anesthesia. d. Intravenous analgesic.

ANS: A Maternal supine hypotension syndrome is caused by the weight and pressure of the gravid uterus on the ascending vena cava when the woman is in a supine position. This reduces venous return to the womans heart, as well as cardiac output, and subsequently reduces her blood pressure. The nurse can encourage the woman to change positions and avoid the supine position. Oxytocin administration, regional anesthesia, and intravenous analgesic may reduce maternal cardiac output. PTS: 1 DIF: Cognitive Level: Comprehension REF: 396 OBJ: Nursing Process: Implementation MSC: Client Needs: Physiologic Integrity

A pregnant woman was admitted for induction of labor at 43 weeks of gestation with sure dates. A nonstress test (NST) in the obstetricians office revealed a nonreactive tracing. On artificial rupture of membranes, thick, meconium-stained fluid was noted. The nurse caring for the infant after birth should anticipate: a. Meconium aspiration, hypoglycemia, and dry, cracked skin. b. Excessive vernix caseosa covering the skin, lethargy, and respiratory distress syndrome. c. Golden yellow- to green stainedskin and nails, absence of scalp hair, and an increased amount of subcutaneous fat. d. Hyperglycemia, hyperthermia, and an alert, wide-eyed appearance.

ANS: A Meconium aspiration, hypoglycemia, and dry, cracked skin are consistent with a postmature infant. Excessive vernix caseosa covering the skin, lethargy, and respiratory distress syndrome would be consistent with a very premature infant. The skin may be meconium stained, but the infant would most likely have longer hair and decreased amounts of subcutaneous fat. Postmaturity with a nonreactive NST would indicate hypoxia. Signs and symptoms associated with fetal hypoxia are hypoglycemia, temperature instability, and lethargy. PTS: 1 DIF: Cognitive Level: Analysis REF: 685 OBJ: Nursing Process: Planning MSC: Client Needs: Physiologic Integrity

Perinatal nurses are legally responsible for: a. Correctly interpreting fetal heart rate (FHR) patterns, initiating appropriate nursing interventions, and documenting the outcomes. b. Greeting the client on arrival, assessing her, and starting an intravenous line. c. Applying the external fetal monitor and notifying the care provider. d. Making sure that the woman is comfortable.

ANS: A Nurses who care for women during childbirth are legally responsible for correctly interpreting FHR patterns, initiating appropriate nursing interventions based on those patterns, and documenting the outcomes of those interventions. Greeting the client, assessing her, and starting an IV; applying the external fetal monitor and notifying the care provider; and making sure the woman is comfortable may be activities that a nurse performs, but they are not activities for which the nurse is legally responsible. PTS: 1 DIF: Cognitive Level: Comprehension REF: 396 OBJ: Nursing Process: Assessment, Planning, Implementation MSC: Client Needs: Safe and Effective Care Environment

A careful review of the literature on the various recreational and illicit drugs reveals that: a. More longer-term studies are needed to assess the lasting effects on infants when mothers have taken or are taking illegal drugs. b. Heroin and methadone cross the placenta; marijuana, cocaine, and phencyclidine (PCP) do not. c. Mothers should discontinue heroin use (detox) any time they can during pregnancy. d. Methadone withdrawal for infants is less severe and shorter than heroin withdrawal.

ANS: A Studies on the effects of marijuana and cocaine use by mothers are somewhat contradictory. More long-range studies are needed. Just about all these drugs cross the placenta, including marijuana, cocaine, and PCP. Drug withdrawal is accompanied by fetal withdrawal, which can lead to fetal death. Therefore, detoxification from heroin is not recommended, particularly later in pregnancy. Methadone withdrawal is more severe and more prolonged than heroin withdrawal. PTS: 1 DIF: Cognitive Level: Analysis REF: 679 OBJ: Nursing Process: Evaluation MSC: Client Needs: Physiologic Integrity

A premature infant with respiratory distress syndrome receives artificial surfactant. How would the nurse explain surfactant therapy to the parents? a. Surfactant improves the ability of your babys lungs to exchange oxygen and carbon dioxide. b. The drug keeps your baby from requiring too much sedation. c. Surfactant is used to reduce episodes of periodic apnea. d. Your baby needs this medication to fight a possible respiratory tract infection.

ANS: A Surfactant can be administered as an adjunct to oxygen and ventilation therapy. With administration of artificial surfactant, respiratory compliance is improved until the infant can generate enough surfactant on his or her own. Surfactant has no bearing on the sedation needs of the infant. Surfactant is used to improve respiratory compliance, including the exchange of oxygen and carbon dioxide. The goal of surfactant therapy in an infant with respiratory distress syndrome (RDS) is to stimulate production of surfactant in the type 2 cells of the alveoli. The clinical presentation of RDS and neonatal pneumonia may be similar. The infant may be started on broad-spectrum antibiotics to treat infection. PTS: 1 DIF: Cognitive Level: Application REF: 687 OBJ: Nursing Process: Planning MSC: Client Needs: Physiologic Integrity

When using intermittent auscultation (IA) to assess uterine activity, the nurse should be cognizant that: a. The examiners hand should be placed over the fundus before, during, and after contractions. b. The frequency and duration of contractions is measured in seconds for consistency. c. Contraction intensity is given a judgment number of 1 to 7 by the nurse and client together. d. The resting tone between contractions is described as either placid or turbulent.

ANS: A The assessment is done by palpation; duration, frequency, intensity, and resting tone must be assessed. The duration of contractions is measured in seconds; the frequency is measured in minutes. The intensity of contractions usually is described as mild, moderate, or strong. The resting tone usually is characterized as soft or relaxed. PTS: 1 DIF: Cognitive Level: Knowledge REF: 385 OBJ: Nursing Process: Assessment MSC: Client Needs: Health Promotion and Maintenance

A pregnant woman presents in labor at term, having had no prenatal care. After birth her infant is noted to be small for gestational age with small eyes and a thin upper lip. The infant also is microcephalic. On the basis of her infants physical findings, this woman should be questioned about her use of which substance during pregnancy? a. Alcohol b. Cocaine c. Heroin d. Marijuana

ANS: A The description of the infant suggests fetal alcohol syndrome, which is consistent with maternal alcohol consumption during pregnancy. Fetal brain, kidney, and urogenital system malformations have been associated with maternal cocaine ingestions. Heroin use in pregnancy frequently results in intrauterine growth restriction. The infant may have a shrill cry and sleep cycle disturbances and present with poor feeding, tachypnea, vomiting, diarrhea, hypothermia or hyperthermia, and sweating. Studies have found a higher incidence of meconium staining in infants born of mothers who used marijuana during pregnancy. PTS: 1 DIF: Cognitive Level: Comprehension REF: 670 OBJ: Nursing Process: Assessment MSC: Client Needs: Physiologic Integrity

In evaluating the effectiveness of oxytocin induction, the nurse would expect: a. Contractions lasting 40 to 90 seconds, 2 to 3 minutes apart. b. The intensity of contractions to be at least 110 to 130 mm Hg. c. Labor to progress at least 2 cm/hr dilation. d. At least 30 mU/min of oxytocin will be needed to achieve cervical dilation.

ANS: A The goal of induction of labor would be to produce contractions that occur every 2 to 3 minutes and last 60 to 90 seconds. The intensity of the contractions should be 40 to 90 mm Hg by intrauterine pressure catheter. Cervical dilation of 1 cm/hr in the active phase of labor would be the goal in an oxytocin induction. The dose is increased by 1 to 2 mU/min at intervals of 30 to 60 minutes until the desired contraction pattern is achieved. Doses are increased up to a maximum of 20 to 40 mU/min. PTS: 1 DIF: Cognitive Level: Analysis REF: 466 OBJ: Nursing Process: Planning MSC: Client Needs: Health Promotion and Maintenance

Immediately after the forceps-assisted birth of an infant, the nurse should: a. Assess the infant for signs of trauma. b. Give the infant prophylactic antibiotics. c. Apply a cold pack to the infants scalp. d. Measure the circumference of the infants head.

ANS: A The infant should be assessed for bruising or abrasions at the site of application, facial palsy, and subdural hematoma. Prophylactic antibiotics are not necessary with a forceps delivery. A cold pack would put the infant at risk for cold stress and is contraindicated. Measuring the circumference of the head is part of the initial nursing assessment. PTS: 1 DIF: Cognitive Level: Application REF: 467 OBJ: Nursing Process: Implementation MSC: Client Needs: Physiologic Integrity

The exact cause of preterm labor is unknown and believed to be multifactorial. Infection is thought to be a major factor in many preterm labors. Select the type of infection that has not been linked to preterm births. a. Viral b. Periodontal c. Cervical d. Urinary tract

ANS: A The infections that increase the risk of preterm labor and birth are all bacterial. They include cervical, urinary tract, periodontal, and other bacterial infections. Therefore, it is important for the client to participate in early, continual, and comprehensive prenatal care. Evidence has shown a link between periodontal infections and preterm labor. Researchers recommend regular dental care before and during pregnancy, oral assessment as a routine part of prenatal care, and scrupulous oral hygiene to prevent infection. Cervical infections of a bacterial nature have been linked to preterm labor and birth. The presence of urinary tract infections increases the risk of preterm labor and birth. PTS: 1 DIF: Cognitive Level: Knowledge REF: 443 OBJ: Nursing Process: Assessment MSC: Client Needs: Physiologic Integrity

In the assessment of a preterm infant, the nurse notices continued respiratory distress even though oxygen and ventilation have been provided. The nurse should suspect: a. Hypovolemia and/or shock. b. A nonneutral thermal environment. c. Central nervous system injury. d. Pending renal failure.

ANS: A The nurse should suspect hypovolemia and/or shock. Other symptoms could include hypotension, prolonged capillary refill, and tachycardia followed by bradycardia. Intervention is necessary. PTS: 1 DIF: Cognitive Level: Application REF: 680 OBJ: Nursing Process: Diagnosis MSC: Client Needs: Physiologic Integrity

The nurse knows that proper placement of the tocotransducer for electronic fetal monitoring is located: a. Over the uterine fundus. b. On the fetal scalp. c. Inside the uterus. d. Over the mothers lower abdomen.

ANS: A The tocotransducer monitors uterine activity and should be placed over the fundus, where the most intensive uterine contractions occur. The tocotransducer is for external use. PTS: 1 DIF: Cognitive Level: Comprehension REF: 386 OBJ: Nursing Process: Implementation MSC: Client Needs: Health Promotion and Maintenance

A pregnant womans amniotic membranes rupture. Prolapsed umbilical cord is suspected. What intervention would be the top priority? a. Placing the woman in the knee-chest position b. Covering the cord in sterile gauze soaked in saline c. Preparing the woman for a cesarean birth d. Starting oxygen by face mask

ANS: A The woman is assisted into a position (e.g., modified Sims position, Trendelenburg position, or the knee-chest position) in which gravity keeps the pressure of the presenting part off the cord. Although covering the cord in sterile gauze soaked saline, preparing the woman for a cesarean, and starting oxygen by face mark are appropriate nursing interventions in the event of a prolapsed cord, the intervention of top priority would be positioning the mother to relieve cord compression. PTS: 1 DIF: Cognitive Level: Application REF: 478 OBJ: Nursing Process: Implementation MSC: Client Needs: Physiologic Integrity

In assisting with the two factors that have an effect on fetal status (i.e., pushing and positioning), nurses should: a. Encourage the womans cooperation in avoiding the supine position. b. Advise the woman to avoid the semi-Fowler position. c. Encourage the woman to hold her breath and tighten her abdominal muscles to produce a vaginal response. d. Instruct the woman to open her mouth and close her glottis, letting air escape after the push.

ANS: A The woman should maintain a side-lying position. The semi-Fowler position is the recommended side-lying position with a lateral tilt to the uterus. The Valsalva maneuver, which encourages the woman to hold her breath and tighten her abdominal muscles, should be avoided. Both the mouth and glottis should be open, letting air escape during the push. PTS: 1 DIF: Cognitive Level: Comprehension REF: 399 OBJ: Nursing Process: Implementation MSC: Client Needs: Health Promotion and Maintenance

During a prenatal examination, the woman reports having two cats at home. The nurse informs her that she should not be cleaning the litter box while she is pregnant. When the woman asks why, the nurses best response would be: a. Your cats could be carrying toxoplasmosis. This is a zoonotic parasite that can infect you and have severe effects on your unborn child. b. You and your baby can be exposed to the human immunodeficiency virus (HIV) in your cats feces. c. Its just gross. You should make your husband clean the litter boxes. d. Cat feces are known to carry Escherichia coli, which can cause a severe infection in both you and your baby.

ANS: A Toxoplasmosis is a multisystem disease caused by the protozoal Toxoplasma gondii parasite, commonly found in cats, dogs, pigs, sheep, and cattle. About 30% of women who contract toxoplasmosis during gestation transmit the disease to their children. Clinical features ascribed to toxoplasmosis include hydrocephalus or microcephaly, chorioretinitis, seizures, or cerebral calcifications. HIV is not transmitted by cats. Although suggesting that the womans husband clean the litter boxes may be a valid statement, it is not appropriate, does not answer the clients question, and is not the nurses best response. E. coli is found in normal human fecal flora. It is not transmitted by cats. PTS: 1 DIF: Cognitive Level: Application REF: 670 OBJ: Nursing Process: Planning MSC: Client Needs: Safe and Effective Care Environment

The most important nursing action in preventing neonatal infection is: a. Good handwashing. b. Isolation of infected infants. c. Separate gown technique. d. Standard Precautions.

ANS: A Virtually all controlled clinical trials have demonstrated that effective handwashing is responsible for the prevention of nosocomial infection in nursery units. Measures to be taken include Standard Precautions, careful and thorough cleaning, frequent replacement of used equipment, and disposal of excrement and linens in an appropriate manner. Overcrowding must be avoided in nurseries. However, the most important nursing action for preventing neonatal infection is effective handwashing. PTS: 1 DIF: Cognitive Level: Comprehension REF: 669 OBJ: Nursing Process: Implementation MSC: Client Needs: Safe and Effective Care Environment

Risk factors associated with necrotizing enterocolitis (NEC) include (Select all that apply): a. Polycythemia. b. Anemia. c. Congenital heart disease. d. Bronchopulmonary dysphasia. e. Retinopathy.

ANS: A, B, C Risk factors for NEC include asphyxia, respiratory distress syndrome, umbilical artery catheterization, exchange transfusion, early enteral feedings, patent ductus arteriosus, congenital heart disease, polycythemia, anemia, shock, and gastrointestinal infection. Bronchopulmonary dysphasia and retinopathy are not associated with NEC. PTS: 1 DIF: Cognitive Level: Comprehension REF: 693 OBJ: Nursing Process: Assessment MSC: Client Needs: Physiologic Integrity

Many common drugs of abuse cause significant physiologic and behavioral problems in infants who are breastfed by mothers currently using (Select all that apply): a. Amphetamine. b. Heroin. c. Nicotine. d. PCP. e. Morphine.

ANS: A, B, C, D Amphetamine, heroin, nicotine, and PCP are contraindicated during breastfeeding because of the reported effects on the infant. Morphine is a medication that often is used to treat neonatal abstinence syndrome. PTS: 1 DIF: Cognitive Level: Comprehension REF: 676 OBJ: Nursing Process: Assessment MSC: Client Needs: Physiologic Integrity

Infants born between 34 0/7 and 36 6/7 weeks of gestation are called late-preterm infants because they have many needs similar to those of preterm infants. Because they are more stable than early-preterm infants, they may receive care that is much like that of a full-term baby. The mother-baby or nursery nurse knows that these babies are at increased risk for (Select all that apply): a. Problems with thermoregulation b. Cardiac distress c. Hyperbilirubinemia d. Sepsis e. Hyperglycemia

ANS: A, C, D Thermoregulation problems, hyperbilirubinemia, and sepsis are all conditions related to immaturity and warrant close observation. After discharge the infant is at risk for rehospitalization related to these problems. AWHONN launched the Near-Term Infant Initiative to study the problem and ways to ensure that these infants receive adequate care. The nurse should ensure that this infant is feeding adequately before discharge and that parents are taught the signs and symptoms of these complications. Late-preterm infants are also at increased risk for respiratory distress and hypoglycemia. PTS: 1 DIF: Cognitive Level: Comprehension REF: 685 OBJ: Nursing Process: Assessment MSC: Client Needs: Health Promotion and Maintenance

Induction of labor is considered an acceptable obstetric procedure if it is in the best interest to deliver the fetus. The charge nurse on the labor and delivery unit is often asked to schedule patients for this procedure and therefore must be cognizant of the specific conditions appropriate for labor induction. These include (Select all that apply): a. Rupture of membranes at or near term. b. Convenience of the woman or her physician. c. Chorioamnionitis (inflammation of the amniotic sac). d. Post-term pregnancy. e. Fetal death.

ANS: A, C, D, E These are all acceptable indications for induction. Other conditions include intrauterine growth retardation (IUGR), maternal-fetal blood incompatibility, hypertension, and placental abruption. Elective inductions for the convenience of the woman or her provider are not recommended; however, they have become commonplace. Factors such as rapid labors and living a long distance from a health care facility may be valid reasons in such a circumstance. Elective delivery should not occur before 39 weeks completed gestation. PTS: 1 DIF: Cognitive Level: Application REF: 461 OBJ: Nursing Process: Planning MSC: Client Needs: Physiologic Integrity

Necrotizing enterocolitis (NEC) is an acute inflammatory disease of the gastrointestinal mucosa that can progress to perforation of the bowel. Approximately 2% to 5% of premature infants succumb to this fatal disease. Care is supportive; however, known interventions may decrease the risk of NEC. To develop an optimal plan of care for this infant, the nurse must understand which intervention has the greatest effect on lowering the risk of NEC: a. Early enteral feedings b. Breastfeeding c. Exchange transfusion d. Prophylactic probiotics

ANS: B A decrease in the incidence of NEC is directly correlated with exclusive breastfeeding. Breast milk enhances maturation of the gastrointestinal tract and contains immune factors that contribute to a lower incidence or severity of NEC, Crohns disease, and celiac illness. The neonatal intensive care unit nurse can be very supportive of the mother in terms of providing her with equipment to pump breast milk, ensuring privacy, and encouraging skin-to-skin contact with the infant. Early enteral feedings of formula or hyperosmolar feedings are a risk factor known to contribute to the development of NEC. The mother should be encouraged to pump or feed breast milk exclusively. Exchange transfusion may be necessary; however, it is a known risk factor for the development of NEC. Although still early, a study in 2005 found that the introduction of prophylactic probiotics appeared to enhance the normal flora of the bowel and therefore decrease the severity of NEC when it did occur. This treatment modality is not as widespread as encouraging breastfeeding; however, it is another strategy that the care providers of these extremely fragile infants may have at their disposal. PTS: 1 DIF: Cognitive Level: Application REF: 693 OBJ: Nursing Process: Planning MSC: Client Needs: Physiologic Integrity

With regard to dysfunctional labor, nurses should be aware that: a. Women who are underweight are more at risk. b. Women experiencing precipitous labor are about the only dysfunctionals not to be exhausted. c. Hypertonic uterine dysfunction is more common than hypotonic dysfunction. d. Abnormal labor patterns are most common in older women.

ANS: B Precipitous labor lasts less than 3 hours. Short women more than 30 pounds overweight are more at risk for dysfunctional labor. Hypotonic uterine dysfunction, in which the contractions become weaker, is more common. Abnormal labor patterns are more common in women less than 20 years of age. PTS: 1 DIF: Cognitive Level: Comprehension REF: 455 OBJ: Nursing Process: Planning MSC: Client Needs: Health Promotion and Maintenance

A maternal indication for the use of vacuum extraction is: a. A wide pelvic outlet. b. Maternal exhaustion. c. A history of rapid deliveries. d. Failure to progress past 0 station.

ANS: B A mother who is exhausted may be unable to assist with the expulsion of the fetus. The patient with a wide pelvic outlet will likely not require vacuum extraction. With a rapid delivery, vacuum extraction is not necessary. A station of 0 is too high for a vacuum extraction. PTS: 1 DIF: Cognitive Level: Knowledge REF: 468 OBJ: Nursing Process: Assessment MSC: Client Needs: Physiologic Integrity

While caring for the patient who requires an induction of labor, the nurse should be cognizant that: a. Ripening the cervix usually results in a decreased success rate for induction. b. Labor sometimes can be induced with balloon catheters or laminaria tents. c. Oxytocin is less expensive than prostaglandins and more effective but creates greater health risks. d. Amniotomy can be used to make the cervix more favorable for labor.

ANS: B Balloon catheters or laminaria tents are mechanical means of ripening the cervix. Ripening the cervix, making it softer and thinner, increases the success rate of induced labor. Prostaglandin E1 is less expensive and more effective than oxytocin but carries a greater risk. Amniotomy is the artificial rupture of membranes, which is used to induce labor only when the cervix is already ripe. PTS: 1 DIF: Cognitive Level: Comprehension REF: 462 OBJ: Nursing Process: Planning MSC: Client Needs: Health Promotion and Maintenance

Nurses should know some basic definitions concerning preterm birth, preterm labor, and low birth weight. For instance: a. The terms preterm birth and low birth weight can be used interchangeably. b. Preterm labor is defined as cervical changes and uterine contractions occurring between 20 and 37 weeks of pregnancy. c. Low birth weight is anything below 3.7 pounds. d. In the United States early in this century, preterm birth accounted for 18% to 20% of all births.

ANS: B Before 20 weeks, it is not viable (miscarriage); after 37 weeks, it can be considered term. Although these terms are used interchangeably, they have different meanings: preterm birth describes the length of gestation (37 weeks) regardless of weight; low birth weight describes weight only (2500 g or less) at the time of birth, whenever it occurs. Low birth weight is anything less than 2500 g, or about 5.5 pounds. In 2003 the preterm birth rate in the United States was 12.3%, but it is increasing in frequency. PTS: 1 DIF: Cognitive Level: Knowledge REF: 441 OBJ: Nursing Process: Assessment MSC: Client Needs: Health Promotion and Maintenance

The goal of treatment of the infant with phenylketonuria (PKU) is to: a. Cure mental retardation. b. Prevent central nervous system (CNS) damage, which leads to mental retardation. c. Prevent gastrointestinal symptoms. d. Cure the urinary tract infection.

ANS: B CNS damage can occur as a result of toxic levels of phenylalanine. No known cure exists for mental retardation. Digestive problems are a clinical manifestation of PKU. PKU does not involve any urinary problems. PTS: 1 DIF: Cognitive Level: Comprehension REF: 711 OBJ: Nursing Process: Planning MSC: Client Needs: Physiologic Integrity

Complications and risks associated with cesarean births include (Select all that apply): a. Placental abruption. b. Wound dehiscence. c. Hemorrhage. d. Urinary tract infections. e. Fetal injuries.

ANS: B, C, D, E Placental abruption and placenta previa are both indications for cesarean birth and are not complications thereof. Wound dehiscence, hemorrhage, urinary tract infection, and fetal injuries are all possible complications and risks associated with delivery by cesarean section. PTS: 1 DIF: Cognitive Level: Comprehension REF: 471 OBJ: Nursing Process: Evaluation MSC: Client Needs: Physiologic Integrity

In planning for an expected cesarean birth for a woman who has given birth by cesarean previously and who has a fetus in the transverse presentation, which information would the nurse include? a. Because this is a repeat procedure, you are at the lowest risk for complications. b. Even though this is your second cesarean birth, you may wish to review the preoperative and postoperative procedures. c. Because this is your second cesarean birth, you will recover faster. d. You will not need preoperative teaching because this is your second cesarean birth.

ANS: B Even though this is your second cesarean birth, you may wish to review the preoperative and postoperative procedures is the most appropriate statement. It is not accurate to state that the woman is at the lowest risk for complications. Both maternal and fetal risks are associated with every cesarean section. Because this is your second cesarean birth, you will recover faster is not an accurate statement. Physiologic and psychologic recovery from a cesarean section is multifactorial and individual to each client each time. Preoperative teaching should always be performed, regardless of whether the client has already had this procedure. PTS: 1 DIF: Cognitive Level: Application REF: 469 OBJ: Nursing Process: Planning MSC: Client Needs: Physiologic Integrity

To care adequately for infants at risk for neonatal bacterial infection, nurses should be aware that: a. Congenital infection progresses more slowly than does nosocomial infection. b. Nosocomial infection can be prevented by effective handwashing; early-onset infections cannot. c. Infections occur with about the same frequency in boy and girl infants, although female mortality is higher. d. The clinical sign of a rapid, high fever makes infection easier to diagnose.

ANS: B Handwashing is an effective preventive measure for late-onset (nosocomial) infections because these infections come from the environment around the infant. Early-onset, or congenital, infections are caused by the normal flora at the maternal vaginal tract and progress more rapidly than do nosocomial (late-onset) infections. Infection occurs about twice as often in boys and results in higher mortality. Clinical signs of neonatal infection are nonspecific and are similar to those of noninfectious problems, thus making diagnosis difficult. PTS: 1 DIF: Cognitive Level: Comprehension REF: 672 OBJ: Nursing Process: Implementation MSC: Client Needs: Safe and Effective Care Environment

The most common cause of pathologic hyperbilirubinemia is: a. Hepatic disease. b. Hemolytic disorders in the newborn. c. Postmaturity. d. Congenital heart defect.

ANS: B Hemolytic disorders in the newborn are the most common cause of pathologic jaundice. Hepatic damage may be a cause of pathologic hyperbilirubinemia, but it is not the most common cause. Prematurity would be a potential cause of pathologic hyperbilirubinemia in neonates, but it is not the most common cause. Congenital heart defect is not a common cause of pathologic hyperbilirubinemia in neonates. PTS: 1 DIF: Cognitive Level: Knowledge REF: 679 OBJ: Nursing Process: Diagnosis MSC: Client Needs: Physiologic Integrity

With regard to small for gestational age (SGA) infants and intrauterine growth restrictions (IUGR), nurses should be aware that: a. In the first trimester diseases or abnormalities result in asymmetric IUGR. b. Infants with asymmetric IUGR have the potential for normal growth and development. c. In asymmetric IUGR weight is slightly more than SGA, whereas length and head circumference are somewhat less than SGA. d. Symmetric IUGR occurs in the later stages of pregnancy.

ANS: B IUGR is either symmetric or asymmetric. The symmetric form occurs in the first trimester; SGA infants have reduced brain capacity. The asymmetric form occurs in the later stages of pregnancy. Weight is less than the 10th percentile; head circumference is greater than the 10th percentile. Infants with asymmetric IUGR have the potential for normal growth and development. PTS: 1 DIF: Cognitive Level: Comprehension REF: 664 OBJ: Nursing Process: Planning MSC: Client Needs: Health Promotion and Maintenance

What correctly matches the type of deceleration with its likely cause? a. Early decelerationumbilical cord compression b. Late decelerationuteroplacental inefficiency c. Variable decelerationhead compression d. Prolonged decelerationcause unknown

ANS: B Late deceleration is caused by uteroplacental inefficiency. Early deceleration is caused by head compression. Variable deceleration is caused by umbilical cord compression. Prolonged deceleration has a variety of either benign or critical causes. PTS: 1 DIF: Cognitive Level: Knowledge REF: 392 OBJ: Nursing Process: Assessment MSC: Client Needs: Physiologic Integrity

To provide optimal care of infants born to mothers who are substance abusers, nurses should be aware that: a. Infants born to addicted mothers are also addicted. b. Mothers who abuse one substance likely will use or abuse another, thus compounding the infants difficulties. c. The NICU Network Neurobehavioral Scale (NNNS) is designed to assess the damage the mother has done to herself. d. No laboratory procedures are available that can identify the intrauterine drug exposure of the infant.

ANS: B Multiple substance use (even just alcohol and tobacco) makes it difficult to assess the problems of the exposed infant, particularly with regard to withdrawal manifestations. Infants of substance-abusing mothers may have some of the physiologic signs but are not addicted in the behavioral sense. Drug-exposed newborn is a more accurate description than addict. The NNNS is designed to assess the neurologic, behavioral, and stress/abstinence function of the neonate. Newborn urine, hair, or meconium sampling may be used to identify an infants intrauterine drug exposure. PTS: 1 DIF: Cognitive Level: Comprehension REF: 676 OBJ: Nursing Process: Evaluation MSC: Client Needs: Psychosocial Integrity

In caring for the preterm infant, what complication is thought to be a result of high arterial blood oxygen level? a. Necrotizing enterocolitis (NEC) b. Retinopathy of prematurity (ROP) c. Bronchopulmonary dysplasia (BPD) d. Intraventricular hemorrhage (IVH)

ANS: B ROP is thought to occur as a result of high levels of oxygen in the blood. NEC is caused by the interference of blood supply to the intestinal mucosa. Necrotic lesions occur at that site. BPD is caused by the use of positive pressure ventilation against the immature lung tissue. IVH results from rupture of the fragile blood vessels in the ventricles of the brain. It is most often associated with hypoxic injury, increased blood pressure, and fluctuating cerebral blood flow. PTS: 1 DIF: Cognitive Level: Comprehension REF: 690 OBJ: Nursing Process: Assessment MSC: Client Needs: Physiologic Integrity

What three measures should the nurse implement to provide intrauterine resuscitation? Select the response that best indicates the priority of actions that should be taken. a. Call the provider, reposition the mother, and perform a vaginal examination. b. Reposition the mother, increase intravenous (IV) fluid, and provide oxygen via face mask. c. Administer oxygen to the mother, increase IV fluid, and notify the care provider. d. Perform a vaginal examination, reposition the mother, and provide oxygen via face mask.

ANS: B Repositioning the mother, increasing intravenous (IV) fluid, and providing oxygen via face mask are correct nursing actions for intrauterine resuscitation. The nurse should initiate intrauterine resuscitation in an ABC manner, similar to basic life support. The first priority is to open the maternal and fetal vascular systems by repositioning the mother for improved perfusion. The second priority is to increase blood volume by increasing the IV fluid. The third priority is to optimize oxygenation of the circulatory volume by providing oxygen via face mask. If these interventions do not resolve the fetal heart rate issue quickly, the primary provider should be notified immediately. PTS: 1 DIF: Cognitive Level: Evaluation REF: 396 OBJ: Nursing Process: Implementation MSC: Client Needs: Health Promotion and Maintenance

Necrotizing enterocolitis (NEC) is an inflammatory disease of the gastrointestinal mucosa. The signs of NEC are nonspecific. Some generalized signs include: a. Hypertonia, tachycardia, and metabolic alkalosis. b. Abdominal distention, temperature instability, and grossly bloody stools. c. Hypertension, absence of apnea, and ruddy skin color. d. Scaphoid abdomen, no residual with feedings, and increased urinary output.

ANS: B Some generalized signs of NEC include decreased activity, hypotonia, pallor, recurrent apnea and bradycardia, decreased oxygen saturation values, respiratory distress, metabolic acidosis, oliguria, hypotension, decreased perfusion, temperature instability, cyanosis, abdominal distention, residual gastric aspirates, vomiting, grossly bloody stools, abdominal tenderness, and erythema of the abdominal wall. The infant may display hypotonia, bradycardia, and metabolic acidosis. PTS: 1 DIF: Cognitive Level: Comprehension REF: 693 OBJ: Nursing Process: Assessment MSC: Client Needs: Physiologic Integrity

The nurse providing care for a woman with preterm labor who is receiving terbutaline would include which intervention to identify side effects of the drug? a. Assessing deep tendon reflexes (DTRs) b. Assessing for chest discomfort and palpitations c. Assessing for bradycardia d. Assessing for hypoglycemia

ANS: B Terbutaline is a b2-adrenergic agonist that affects the cardiopulmonary and metabolic systems of the mother. Signs of cardiopulmonary decompensation would include chest pain and palpitations. Assessing DTRs would not address these concerns. b2-Adrenergic agonist drugs cause tachycardia, not bradycardia. The metabolic effect leads to hyperglycemia, not hypoglycemia. PTS: 1 DIF: Cognitive Level: Analysis REF: 447 OBJ: Nursing Process: Assessment MSC: Client Needs: Physiologic Integrity

During labor a fetus with an average heart rate of 135 beats/min over a 10-minute period would be considered to have: a. Bradycardia. b. A normal baseline heart rate. c. Tachycardia. d. Hypoxia.

ANS: B The baseline heart rate is measured over 10 minutes; a normal range is 110 to 160 beats/min. Bradycardia is a fetal heart rate (FHR) below 110 beats/min for 10 minutes or longer. Tachycardia is an FHR over 160 beats/min for 10 minutes or longer. Hypoxia is an inadequate supply of oxygen; no indication of this condition exists with a baseline heart rate in the normal range. PTS: 1 DIF: Cognitive Level: Knowledge REF: 389 OBJ: Nursing Process: Assessment MSC: Client Needs: Physiologic Integrity

The nurse caring for the woman in labor should understand that increased variability of the fetal heart rate may be caused by: a. Narcotics. b. Barbiturates. c. Methamphetamines. d. Tranquilizers.

ANS: C Narcotics, barbiturates, and tranquilizers may be causes of decreased variability; methamphetamines may cause increased variability. PTS: 1 DIF: Cognitive Level: Comprehension REF: 390 OBJ: Nursing Process: Assessment MSC: Client Needs: Health Promotion and Maintenance

A pregnant woman at 29 weeks of gestation has been diagnosed with preterm labor. Her labor is being controlled with tocolytic medications. She asks when she would be able to go home. Which response by the nurse is most accurate? a. After the baby is born. b. When we can stabilize your preterm labor and arrange home health visits. c. Whenever the doctor says that it is okay. d. It depends on what kind of insurance coverage you have.

ANS: B The clients preterm labor is being controlled with tocolytics. Once she is stable, home care may be a viable option for this type of client. Care of a woman with preterm labor is multifactorial; the goal is to prevent delivery. In many cases this may be achieved at home. Care of the preterm client is multidisciplinary and multifactorial. Managed care may dictate earlier hospital discharges or a shift from hospital to home care. Insurance coverage may be one factor in the care of clients, but ultimately client safety remains the most important factor. PTS: 1 DIF: Cognitive Level: Application REF: 447 OBJ: Nursing Process: Planning MSC: Client Needs: Health Promotion and Maintenance

A newborn was admitted to the neonatal intensive care unit after being delivered at 29 weeks of gestation to a 28-year-old multiparous, married, Caucasian woman whose pregnancy was uncomplicated until premature rupture of membranes and preterm birth. The newborns parents arrive for their first visit after the birth. The parents walk toward the bedside but remain approximately 5 feet away from the bed. The nurses most appropriate action would be to: a. Wait quietly at the newborns bedside until the parents come closer. b. Go to the parents, introduce himself or herself, and gently encourage the parents to come meet their infant; explain the equipment first, and then focus on the newborn. c. Leave the parents at the bedside while they are visiting so they can have some privacy. d. Tell the parents only about the newborns physical condition, and caution them to avoid touching their baby.

ANS: B The nurse is instrumental in the initial interactions with the infant. The nurse can help the parents see the infant, rather than focus on the equipment. The importance and purpose of the apparatus that surrounds their infant also should be explained to them. Parents often need encouragement and recognition from the nurse to acknowledge the reality of the infants condition. Parents need to see and touch their infant as soon as possible to acknowledge the reality of the birth and the infants appearance and condition. Encouragement from the nurse is instrumental in this process. Telling the parents only about the newborns physical condition and cautioning them to avoid touching their baby is an inappropriate action. PTS: 1 DIF: Cognitive Level: Application REF: 705 OBJ: Nursing Process: Implementation MSC: Client Needs: Psychosocial Integrity

A new client and her partner arrive on the labor, delivery, recovery, and postpartum unit for the birth of their first child. You apply the electronic fetal monitor (EFM) to the woman. Her partner asks you to explain what is printing on the graph, referring to the EFM strip. He wants to know what the babys heart rate should be. Your best response is: a. Dont worry about that machine; thats my job. b. The top line graphs the babys heart rate. Generally the heart rate is between 110 and 160. The heart rate will fluctuate in response to what is happening during labor. c. The top line graphs the babys heart rate, and the bottom line lets me know how strong the contractions are. d. Your doctor will explain all of that later.

ANS: B The top line graphs the babys heart rate. Generally the heart rate is between 110 and 160. The heart rate will fluctuate in response to what is happening during labor educates the partner about fetal monitoring and provides support and information to alleviate his fears. Dont worry about that machine; thats my job discredits the partners feelings and does not provide the teaching he is requesting. The top line graphs the babys heart rate, and the bottom line lets me know how strong the contractions are provides inaccurate information and does not address the partners concerns about the fetal heart rate. The EFM graphs the frequency and duration of the contractions, not the intensity. Nurses should take every opportunity to provide client and family teaching, especially when information is requested. PTS: 1 DIF: Cognitive Level: Application REF: 397 OBJ: Nursing Process: Planning MSC: Client Needs: Psychosocial Integrity

Why is continuous electronic fetal monitoring usually used when oxytocin is administered? a. The mother may become hypotensive. b. Uteroplacental exchange may be compromised. c. Maternal fluid volume deficit may occur. d. Fetal chemoreceptors are stimulated.

ANS: B The uterus may contract more firmly, and the resting tone may be increased with oxytocin use. This response reduces entrance of freshly oxygenated maternal blood into the intervillous spaces, thus depleting fetal oxygen reserves. Hypotension is not a common side effect of oxytocin. All laboring women are at risk for fluid volume deficit; oxytocin administration does not increase the risk. Oxytocin affects the uterine muscles. PTS: 1 DIF: Cognitive Level: Comprehension REF: 382 OBJ: Nursing Process: Planning MSC: Client Needs: Physiologic Integrity

The nurse is caring for a client whose labor is being augmented with oxytocin. He or she recognizes that the oxytocin should be discontinued immediately if there is evidence of: a. Uterine contractions occurring every 8 to 10 minutes. b. A fetal heart rate (FHR) of 180 with absence of variability. c. The clients needing to void. d. Rupture of the clients amniotic membranes.

ANS: B This FHR is nonreassuring. The oxytocin should be discontinued immediately, and the physician should be notified. The oxytocin should be discontinued if uterine hyperstimulation occurs. Uterine contractions that are occurring every 8 to 10 minutes do not qualify as hyperstimulation. The clients needing to void is not an indication to discontinue the oxytocin induction immediately or to call the physician. Unless a change occurs in the FHR pattern that is nonreassuring or the client experiences uterine hyperstimulation, the oxytocin does not need to be discontinued. The physician should be notified that the clients membranes have ruptured. PTS: 1 DIF: Cognitive Level: Evaluation REF: 449 OBJ: Nursing Process: Planning MSC: Client Needs: Physiologic Integrity

The nurse providing care for the laboring woman realizes that variable fetal heart rate (FHR) decelerations are caused by: a. Altered fetal cerebral blood flow. b. Umbilical cord compression. c. Uteroplacental insufficiency. d. Fetal hypoxemia.

ANS: B Variable decelerations can occur any time during the uterine contracting phase and are caused by compression of the umbilical cord. Altered fetal cerebral blood flow would result in early decelerations in the FHR. Uteroplacental insufficiency would result in late decelerations in the FHR. Fetal hypoxemia would result in tachycardia initially and then bradycardia if hypoxia continues. PTS: 1 DIF: Cognitive Level: Knowledge REF: 393 OBJ: Nursing Process: Assessment MSC: Client Needs: Physiologic Integrity

A tiered system of categorizing FHR has been recommended by regulatory agencies. Nurses, midwives, and physicians who care for women in labor must have a working knowledge of fetal monitoring standards and understand the significance of each category. These categories include (Select all that apply): a. Reassuring. b. Category I. c. Category II. d. Nonreassuring. e. Category III.

ANS: B, C, E The three tiered system of FHR tracings include Category I, II, and III. Category I is a normal tracing requiring no action. Category II FHR tracings are indeterminate. This category includes tracings that do not meet Category I or III criteria. Category III tracings are abnormal and require immediate intervention. PTS: 1 DIF: Cognitive Level: Comprehension REF: 396 OBJ: Nursing Process: Assessment, Planning MSC: Client Needs: Physiologic Integrity

The nurse recognizes that uterine hyperstimulation with oxytocin requires emergency interventions. What clinical cues would alert the nurse that the woman is experiencing uterine hyperstimulation (Select all that apply)? a. Uterine contractions lasting <90 seconds and occurring >2 minutes in frequency b. Uterine contractions lasting >90 seconds and occurring <2 minutes in frequency c. Uterine tone <20 mm Hg d. Uterine tone >20 mm Hg e. Increased uterine activity accompanied by a nonreassuring fetal heart rate (FHR) and pattern

ANS: B, D, E Uterine contractions that occur less than 2 minutes apart and last more than 90 seconds, a uterine tone of over 20 mm Hg, and a nonreassuring FHR and pattern are all indications of uterine hyperstimulation with oxytocin administration. Uterine contractions that occur more than 2 minutes apart and last less than 90 seconds are the expected goal of oxytocin induction. A uterine tone of less than 20 mm Hg is normal. PTS: 1 DIF: Cognitive Level: Analysis REF: 464 OBJ: Nursing Process: Implementation MSC: Client Needs: Physiologic Integrity

As related to central nervous system injuries that could occur to the infant during labor and birth, nurses should be aware that: a. Intracranial hemorrhage (ICH) as a result of birth trauma is more likely to occur in the preterm, low-birth-weight infant. b. Subarachnoid hemorrhage (the most common form of ICH) occurs in term infants as a result of hypoxia. c. In many infants signs of hemorrhage in a full-term infant are absent and are diagnosed only through laboratory tests. d. Spinal cord injuries almost always result from forceps-assisted deliveries.

ANS: C Abnormalities in lumbar punctures or red blood cell counts, for instance, or in visuals on computed tomography scan may reveal a hemorrhage. ICH as a result of birth trauma is more likely to occur in the fullterm, large infant. Subarachnoid hemorrhage in term infants is a result of trauma; in preterm infants it is a result of hypoxia. Spinal cord injuries are almost always from breech births; they are rare today because cesarean birth often is used for breech presentation. PTS: 1 DIF: Cognitive Level: Comprehension REF: 666 OBJ: Nursing Process: Planning MSC: Client Needs: Physiologic Integrity

The priority nursing care associated with an oxytocin (Pitocin) infusion is: a. Measuring urinary output. b. Increasing infusion rate every 30 minutes. c. Monitoring uterine response. d. Evaluating cervical dilation.

ANS: C Because of the risk of hyperstimulation, which could result in decreased placental perfusion and uterine rupture, the nurses priority intervention is monitoring uterine response. Monitoring urinary output is also important; however, it is not the top priority during the administration of Pitocin. The infusion rate may be increased after proper assessment that it is an appropriate interval to do so. Monitoring labor progression is the standard of care for all labor patients. PTS: 1 DIF: Cognitive Level: Comprehension REF: 464 OBJ: Nursing Process: Implementation MSC: Client Needs: Physiologic Integrity

As relates to the use of tocolytic therapy to suppress uterine activity, nurses should be aware that: a. The drugs can be given efficaciously up to the designated beginning of term at 37 weeks. b. There are no important maternal (as opposed to fetal) contraindications. c. Its most important function is to afford the opportunity to administer antenatal glucocorticoids. d. If the client develops pulmonary edema while receiving tocolytics, intravenous (IV) fluids should be given.

ANS: C Buying time for antenatal glucocorticoids to accelerate fetal lung development may be the best reason to use tocolytics. Once the pregnancy has reached 34 weeks, the risks of tocolytic therapy outweigh the benefits. There are important maternal contraindications to tocolytic therapy. Tocolytic-induced edema can be caused by IV fluids. PTS: 1 DIF: Cognitive Level: Comprehension REF: 450 OBJ: Nursing Process: Planning MSC: Client Needs: Physiologic Integrity

An infant is to receive gastrostomy feedings. What intervention should the nurse institute to prevent bloating, gastrointestinal reflux into the esophagus, vomiting, and respiratory compromise? a. Rapid bolusing of the entire amount in 15 minutes b. Warm cloths to the abdomen for the first 10 minutes c. Slow, small, warm bolus feedings over 30 minutes d. Cold, medium bolus feedings over 20 minutes

ANS: C Feedings by gravity are done slowly over 20- to 30-minute periods to prevent adverse reactions. Rapid bolusing of the entire amount in 15 minutes would most likely lead to the adverse reactions listed. Temperature stability in the newborn is critical. Warm cloths to the abdomen for the first 10 minutes would not be appropriate because it is not a thermoregulated environment. Additionally, abdominal warming is not indicated with feedings of any kind. Small feedings at room temperature are recommended to prevent adverse reactions. PTS: 1 DIF: Cognitive Level: Application REF: 700 OBJ: Nursing Process: Implementation MSC: Client Needs: Physiologic Integrity

Fetal bradycardia is most common during: a. Intraamniotic infection. b. Fetal anemia. c. Prolonged umbilical cord compression. d. Tocolytic treatment using terbutaline.

ANS: C Fetal bradycardia can be considered a later sign of fetal hypoxia and is known to occur before fetal death. Bradycardia can result from placental transfer of drugs, prolonged compression of the umbilical cord, maternal hypothermia, and maternal hypotension. Intraamniotic infection, fetal anemia, and tocolytic treatment using terbutaline would most likely result in fetal tachycardia. PTS: 1 DIF: Cognitive Level: Comprehension REF: 390 OBJ: Nursing Process: Assessment MSC: Client Needs: Physiologic Integrity

As related to the eventual discharge of the high risk newborn or transfer to a different facility, nurses and families should be aware that: a. Infants will stay in the neonatal intensive care unit (NICU) until they are ready to go home. b. Once discharged to home, the high risk infant should be treated like any healthy term newborn. c. Parents of high risk infants need special support and detailed contact information. d. If a high risk infant and mother need transfer to a specialized regional center, it is better to wait until after birth and the infant is stabilized.

ANS: C High risk infants can cause profound parental stress and emotional turmoil. Parents need support, special teaching, and quick access to various resources available to help them care for their baby. Parents and their high risk infant should spend a night or two in a predischarge room, where care for the infant is provided away from the NICU. Just because high risk infants are discharged does not mean that they are normal, healthy babies. Follow-up by specialized practitioners is essential. Ideally, the mother and baby are transported with the fetus in utero; this reduces neonatal morbidity and mortality. PTS: 1 DIF: Cognitive Level: Comprehension REF: 664 OBJ: Nursing Process: Planning MSC: Client Needs: Psychosocial Integrity

An infant was born 2 hours ago at 37 weeks of gestation and weighing 4.1 kg. The infant appears chubby with a flushed complexion and is very tremulous. The tremors are most likely the result of: a. Birth injury. b. Hypocalcemia. c. Hypoglycemia. d. Seizures.

ANS: C Hypoglycemia is common in the macrosomic infant. Signs of hypoglycemia include jitteriness, apnea, tachypnea, and cyanosis. PTS: 1 DIF: Cognitive Level: Comprehension REF: 682 OBJ: Nursing Process: Assessment MSC: Client Needs: Physiologic Integrity

Infants of mothers with diabetes (IDMs) are at higher risk for developing: a. Anemia. b. Hyponatremia. c. Respiratory distress syndrome. d. Sepsis.

ANS: C IDMs are at risk for macrosomia, birth injury, perinatal asphyxia, respiratory distress syndrome, hypoglycemia, hypocalcemia, hypomagnesemia, cardiomyopathy, hyperbilirubinemia, and polycythemia. They are not at risk for anemia, hyponatremia, or sepsis. PTS: 1 DIF: Cognitive Level: Comprehension REF: 682 OBJ: Nursing Process: Planning MSC: Client Needs: Physiologic Integrity

Prepidil (prostaglandin gel) has been ordered for a pregnant woman at 43 weeks of gestation. The nurse recognizes that this medication will be administered to: a. Enhance uteroplacental perfusion in an aging placenta. b. Increase amniotic fluid volume. c. Ripen the cervix in preparation for labor induction . d. Stimulate the amniotic membranes to rupture.

ANS: C It is accurate to state that Prepidil will be administered to ripen the cervix in preparation for labor induction. It is not administered to enhance uteroplacental perfusion in an aging placenta, increase amniotic fluid volume, or stimulate the amniotic membranes to rupture. PTS: 1 DIF: Cognitive Level: Application REF: 463 OBJ: Nursing Process: Planning MSC: Client Needs: Physiologic Integrity

The nurse practicing in the perinatal setting should promote kangaroo care regardless of an infants gestational age. This intervention: a. Is adopted from classical British nursing traditions. b. Helps infants with motor and central nervous system impairment. c. Helps infants to interact directly with their parents and enhances their temperature regulation. d. Gets infants ready for breastfeeding.

ANS: C Kangaroo care is skin-to-skin holding in which the infant, dressed only in a diaper, is placed directly on the parents bare chest and then covered. The procedure helps infants interact with their parents and regulates their temperature, among other developmental benefits. PTS: 1 DIF: Cognitive Level: Knowledge REF: 699 OBJ: Nursing Process: Implementation MSC: Client Needs: Health Promotion and Maintenance

The nurse caring for the woman in labor should understand that maternal hypotension can result in: a. Early decelerations. b. Fetal dysrhythmias. c. Uteroplacental insufficiency. d. Spontaneous rupture of membranes.

ANS: C Low maternal blood pressure reduces placental blood flow during uterine contractions and results in fetal hypoxemia. Maternal hypotension is not associated with early decelerations, fetal dysrhythmias, or spontaneous rupture of membranes. PTS: 1 DIF: Cognitive Level: Comprehension REF: 394 OBJ: Nursing Process: Assessment MSC: Client Needs: Health Promotion and Maintenance

The nurse knows that a baby born to a mother who had oligohydramnios could show signs of which of the following? 1. Respiratory difficulty 2. Hypertension 3. Heart murmur 4. Decreased temperature

Answer: 1 Explanation: 1. Because there is less fluid available for the fetus to use during fetal breathing movements, pulmonary hypoplasia may develop.

Providing care for the neonate born to a mother who abuses substances can present a challenge for the health care team. Nursing care for this infant requires a multisystem approach. The first step in the provision of this care is: a. Pharmacologic treatment. b. Reduction of environmental stimuli. c. Neonatal abstinence syndrome scoring. d. Adequate nutrition and maintenance of fluid and electrolyte balance.

ANS: C Neonatal abstinence syndrome (NAS) is the term used to describe the cohort of symptoms associated with drug withdrawal in the neonate. The Neonatal Abstinence Scoring System evaluates central nervous system (CNS), metabolic, vasomotor, respiratory, and gastrointestinal disturbances. This evaluation tool enables the care team to develop an appropriate plan of care. The infant is scored throughout the length of stay, and the treatment plan is adjusted accordingly. Pharmacologic treatment is based on the severity of withdrawal symptoms. Symptoms are determined by using a standard assessment tool. Medications of choice are morphine, phenobarbital, diazepam, or diluted tincture of opium. Swaddling, holding, and reducing environmental stimuli are essential in providing care to the infant who is experiencing withdrawal. These nursing interventions are appropriate for the infant who displays CNS disturbances. Poor feeding is one of the gastrointestinal symptoms common to this client population. Fluid and electrolyte balance must be maintained and adequate nutrition provided. These infants often have a poor suck reflex and may need to be fed via gavage. PTS: 1 DIF: Cognitive Level: Application REF: 670 OBJ: Nursing Process: Assessment MSC: Client Needs: Physiologic Integrity

Surgical, medical, or mechanical methods may be used for labor induction. Which technique is considered a mechanical method of induction? a. Amniotomy b. Intravenous Pitocin c. Transcervical catheter d. Vaginal insertion of prostaglandins

ANS: C Placement of a balloon-tipped Foley catheter into the cervix is a mechanical method of induction. Other methods to expand and gradually dilate the cervix include hydroscopic dilators such as laminaria tents (made from desiccated seaweed), or Lamicel (contains magnesium sulfate). Amniotomy is a surgical method of augmentation and induction. Intravenous Pitocin and insertion of prostaglandins are medical methods of induction. PTS: 1 DIF: Cognitive Level: Application REF: 462 OBJ: Nursing Process: Implementation MSC: Client Needs: Physiologic Integrity

For clinical purposes, preterm and post-term infants are defined as: a. Preterm before 34 weeks if appropriate for gestational age (AGA) and before 37 weeks if small for gestational age (SGA). b. Post-term after 40 weeks if large for gestational age (LGA) and beyond 42 weeks if AGA. c. Preterm before 37 weeks, and post-term beyond 42 weeks, no matter the size for gestational age at birth. d. Preterm, SGA before 38 to 40 weeks, and post-term, LGA beyond 40 to 42 weeks.

ANS: C Preterm and post-term are strictly measures of timebefore 37 weeks and beyond 42 weeks, respectivelyregardless of size for gestational age. PTS: 1 DIF: Cognitive Level: Comprehension REF: 664 OBJ: Nursing Process: Diagnosis MSC: Client Needs: Health Promotion and Maintenance

In planning for home care of a woman with preterm labor, which concern must the nurse address? a. Nursing assessments will be different from those done in the hospital setting. b. Restricted activity and medications will be necessary to prevent recurrence of preterm labor. c. Prolonged bed rest may cause negative physiologic effects. d. Home health care providers will be necessary.

ANS: C Prolonged bed rest may cause adverse effects such as weight loss, loss of appetite, muscle wasting, weakness, bone demineralization, decreased cardiac output, risk for thrombophlebitis, alteration in bowel functions, sleep disturbance, and prolonged postpartum recovery. Nursing assessments will differ somewhat from those performed in the acute care setting, but this is not the concern that needs to be addressed. Restricted activity and medication may prevent preterm labor, but not in all women. In addition, the plan of care is individualized to meet the needs of each woman. Many women will receive home health nurse visits, but care is individualized for each woman. PTS: 1 DIF: Cognitive Level: Analysis REF: 446 OBJ: Nursing Process: Planning MSC: Client Needs: Health Promotion and Maintenance

The nurse caring for a woman in labor understands that prolonged decelerations: a. Are a continuing pattern of benign decelerations that do not require intervention. b. Constitute a baseline change when they last longer than 5 minutes. c. Usually are isolated events that end spontaneously. d. Require the usual fetal monitoring by the nurse.

ANS: C Prolonged decelerations usually are isolated events that end spontaneously. However, in certain combinations with late and/or variable decelerations, they are a danger sign that requires the nurse to notify the physician or midwife immediately. A deceleration that lasts longer than 10 minutes constitutes a baseline change. PTS: 1 DIF: Cognitive Level: Knowledge REF: 393 OBJ: Nursing Process: Assessment MSC: Client Needs: Health Promotion and Maintenance

In caring for the mother who has abused (or is abusing) alcohol and for her infant, nurses should be aware that: a. The pattern of growth restriction of the fetus begun in prenatal life is halted after birth, and normal growth takes over. b. Two thirds of newborns with fetal alcohol syndrome (FAS) are boys. c. Alcohol-related neurodevelopmental disorders not sufficient to meet FAS criteria (learning disabilities, speech and language problems) are often not detected until the child goes to school. d. Both the distinctive facial features of the FAS infant and the diminished mental capacities tend toward normal over time.

ANS: C Some learning problems do not become evident until the child is at school. The pattern of growth restriction persists after birth. Two thirds of newborns with FAS are girls. Although the distinctive facial features of the FAS infant tend to become less evident, the mental capacities never become normal. PTS: 1 DIF: Cognitive Level: Comprehension REF: 676 OBJ: Nursing Process: Planning MSC: Client Needs: Health Promotion and Maintenance

According to standard professional thinking, nurses should auscultate the fetal heart rate (FHR): a. Every 15 minutes in the active phase of the first stage of labor in the absence of risk factors. b. Every 20 minutes in the second stage, regardless of whether risk factors are present. c. Before and after ambulation and rupture of membranes. d. More often in a womans first pregnancy.

ANS: C The FHR should be auscultated before and after administration of medications and induction of anesthesia. In the active phase of the first stage of labor, the FHR should be auscultated every 30 minutes if no risk factors are involved; with risk factors it should be auscultated every 15 minutes. In the second stage of labor the FHR should be auscultated every 15 minutes if no risk factors are involved; with risk factors it should be auscultated every 5 minutes. The fetus of a first-time mother is automatically at greater risk. PTS: 1 DIF: Cognitive Level: Comprehension REF: 385 OBJ: Nursing Process: Implementation MSC: Client Needs: Health Promotion and Maintenance

An infant is being discharged from the neonatal intensive care unit after 70 days of hospitalization. The infant was born at 30 weeks of gestation with several conditions associated with prematurity, including respiratory distress syndrome, mild bronchopulmonary dysplasia, and retinopathy of prematurity requiring surgical treatment. During discharge teaching the infants mother asks the nurse whether her baby will meet developmental milestones on time, as did her son who was born at term. The nurses most appropriate response is: a. Your baby will develop exactly like your first child did. b. Your baby does not appear to have any problems at the present time. c. Your baby will need to be corrected for prematurity. Your baby is currently 40 weeks of postconceptional age and can be expected to be doing what a 40-week-old infant would be doing. d. Your baby will need to be followed very closely.

ANS: C The age of a preterm newborn is corrected by adding the gestational age and the postnatal age. The infants responses are evaluated accordingly against the norm expected for the corrected age of the infant. Although it is impossible to predict with complete accuracy the growth and development potential of each preterm infant, certain measurable factors predict normal growth and development. The preterm infant experiences catch-up body growth during the first 2 to 3 years of life. The growth and developmental milestones are corrected for gestational age until the child is approximately 2.5 years old. Stating that the baby does not appear to have any problems at the present time is inaccurate. Development will need to be evaluated over time. PTS: 1 DIF: Cognitive Level: Application REF: 676 OBJ: Nursing Process: Planning MSC: Client Needs: Health Promotion and Maintenance

To provide safe care for the woman, the nurse understands that which condition is a contraindication for an amniotomy? a. Dilation less than 3 cm b. Cephalic presentation c. -2 station d. Right occiput posterior position

ANS: C The dilation of the cervix must be great enough to determine labor. The presenting part of the fetus should be engaged and well applied to the cervix before the procedure in order to prevent cord prolapse. Amniotomy is deferred if the presenting part is higher in the pelvis. ROP indicates a cephalic presentation, which is appropriate for an amniotomy. PTS: 1 DIF: Cognitive Level: Comprehension REF: 464 OBJ: Nursing Process: Assessment MSC: Client Needs: Physiologic Integrity

When assessing the preterm infant the nurse understands that compared with the term infant, the preterm infant has: a. Few blood vessels visible through the skin. b. More subcutaneous fat. c. Well-developed flexor muscles. d. Greater surface area in proportion to weight.

ANS: D Preterm infants have greater surface area in proportion to their weight. More subcutaneous fat and welldeveloped muscles are indications of a more mature infant. PTS: 1 DIF: Cognitive Level: Analysis REF: 685 OBJ: Nursing Process: Assessment MSC: Client Needs: Physiologic Integrity

As a result of large body surface in relation to weight, the preterm infant is at high risk for heat loss and cold stress. By understanding the four mechanisms of heat transfer (convection, conduction, radiation, and evaporation), the nurse can create an environment for the infant that prevents temperature instability. While evaluating the plan that has been implemented, the nurse knows that the infant is experiencing cold stress when he or she exhibits: a. Decreased respiratory rate. b. Bradycardia followed by an increased heart rate. c. Mottled skin with acrocyanosis. d. Increased physical activity.

ANS: C The infant has minimal to no fat stores. During times of cold stress the skin will become mottled, and acrocyanosis will develop, progressing to cyanosis. Even if the infant is being cared for on a radiant warmer or in an isolette, the nurses role is to observe the infant frequently to prevent heat loss and respond quickly if signs and symptoms occur. The respiratory rate increases followed by periods of apnea. The infant initially tries to conserve heat and burns more calories, after which the metabolic system goes into overdrive. In the preterm infant experiencing heat loss, the heart rate initially increases, followed by periods of bradycardia. In the term infant, the natural response to heat loss is increased physical activity. However, in a term infant experiencing respiratory distress or in a preterm infant, physical activity is decreased. PTS: 1 DIF: Cognitive Level: Analysis REF: 696 OBJ: Nursing Process: Evaluation MSC: Client Needs: Physiologic Integrity

A plan of care for an infant experiencing symptoms of drug withdrawal should include: a. Administering chloral hydrate for sedation. b. Feeding every 4 to 6 hours to allow extra rest. c. Swaddling the infant snugly and holding the baby tightly. d. Playing soft music during feeding.

ANS: C The infant should be wrapped snugly to reduce self-stimulation behaviors and protect the skin from abrasions. Phenobarbital or diazepam may be administered to decrease central nervous system (CNS) irritability. The infant should be fed in small, frequent amounts and burped well to diminish aspiration and maintain hydration. The infant should not be stimulated (such as with music) because this will increase activity and potentially increase CNS irritability. PTS: 1 DIF: Cognitive Level: Application REF: 675 OBJ: Nursing Process: Implementation MSC: Client Needs: Physiologic Integrity

The least common cause of long, difficult, or abnormal labor (dystocia) is: a. Midplane contracture of the pelvis. b. Compromised bearing-down efforts as a result of pain medication. c. Disproportion of the pelvis. d. Low-lying placenta.

ANS: C The least common cause of dystocia is disproportion of the pelvis. PTS: 1 DIF: Cognitive Level: Knowledge REF: 455 OBJ: Nursing Process: Planning MSC: Client Needs: Health Promotion and Maintenance

The nurse providing care to a woman in labor should understand that cesarean birth: a. Is declining in frequency in the twenty-first century in the United States. b. Is more likely to be performed for poor women in public hospitals who do not receive the nurse counseling as do wealthier clients. c. Is performed primarily for the benefit of the fetus. d. Can be either elected or refused by women as their absolute legal right.

ANS: C The most common indications for cesarean birth are danger to the fetus related to labor and birth complications. Cesarean births are increasing in the United States in this century. Wealthier women who have health insurance and who give birth in a private hospital are more likely to experience cesarean birth. A womans right to elect cesarean surgery is in dispute, as is her right to refuse it if in doing so she endangers the fetus. Legal issues are not absolutely clear. PTS: 1 DIF: Cognitive Level: Comprehension REF: 471 OBJ: Nursing Process: Planning MSC: Client Needs: Physiologic Integrity

The uterine contractions of a woman early in the active phase of labor are assessed by an internal uterine pressure catheter (IUPC). The nurse notes that the intrauterine pressure at the peak of the contraction ranges from 65 to 70 mm Hg and the resting tone range is 6 to 10 mm Hg. The uterine contractions occur every 3 to 4 minutes and last an average of 55 to 60 seconds. On the basis of this information, the nurse should: a. Notify the womans primary health care provider immediately. b. Prepare to administer an oxytocic to stimulate uterine activity. c. Document the findings because they reflect the expected contraction pattern for the active phase of labor. d. Prepare the woman for the onset of the second stage of labor.

ANS: C The nurse is responsible for monitoring the uterine contractions to ascertain whether they are powerful and frequent enough to accomplish the work of expelling the fetus and the placenta. In addition, the nurse would document these findings in the clients medical record. This labor pattern indicates that the client is in the active phase of the first stage of labor. Nothing indicates a need to notify the primary care provider at this time. Oxytocin augmentation is not needed for this labor pattern; this contraction pattern indicates adequate active labor. Her contractions eventually will become stronger, last longer, and come closer together during the transition phase of the first stage of labor. The transition phase precedes the second stage of labor, or delivery of the fetus. PTS: 1 DIF: Cognitive Level: Application REF: 399 OBJ: Nursing Process: Implementation MSC: Client Needs: Health Promotion and Maintenance

Near the end of the first week of life, an infant who has not been treated for any infection develops a copper-colored, maculopapular rash on the palms and around the mouth and anus. The newborn is showing signs of: a. Gonorrhea. b. Herpes simplex virus infection. c. Congenital syphilis. d. Human immunodeficiency virus.

ANS: C The rash is indicative of congenital syphilis. The lesions may extend over the trunk and extremities. PTS: 1 DIF: Cognitive Level: Analysis REF: 673 OBJ: Nursing Process: Diagnosis MSC: Client Needs: Physiologic Integrity

What is an advantage of external electronic fetal monitoring? a. The ultrasound transducer can accurately measure short-term variability and beat-to-beat changes in the fetal heart rate. b. The tocotransducer can measure and record the frequency, regularity, intensity, and approximate duration of uterine contractions (UCs). c. The tocotransducer is especially valuable for measuring uterine activity during the first stage of labor. d. Once correctly applied by the nurse, the transducer need not be repositioned even when the woman changes positions.

ANS: C The tocotransducer is especially valuable for measuring uterine activity during the first stage of labor, particularly when the membranes are intact. Short-term changes cannot be measured with this technology. The tocotransducer cannot measure and record the intensity of UCs. The transducer must be repositioned when the woman or fetus changes position. PTS: 1 DIF: Cognitive Level: Comprehension REF: 385 OBJ: Nursing Process: Implementation MSC: Client Needs: Health Promotion and Maintenance

The nurse providing care for the laboring woman should understand that late fetal heart rate (FHR) decelerations are the result of: a. Altered cerebral blood flow. b. Umbilical cord compression. c. Uteroplacental insufficiency. d. Meconium fluid.

ANS: C Uteroplacental insufficiency would result in late decelerations in the FHR. Altered fetal cerebral blood flow would result in early decelerations in the FHR. Umbilical cord compression would result in variable decelerations in the FHR. Meconium-stained fluid may or may not produce changes in the fetal heart rate, depending on the gestational age of the fetus and whether other causative factors associated with fetal distress are present. PTS: 1 DIF: Cognitive Level: Knowledge REF: 398 OBJ: Nursing Process: Assessment MSC: Client Needs: Physiologic Integrity

A woman is having her first child. She has been in labor for 15 hours. Two hours ago her vaginal examination revealed the cervix to be dilated to 5 cm and 100% effaced, and the presenting part was at station 0. Five minutes ago her vaginal examination indicated that there had been no change. What abnormal labor pattern is associated with this description? a. Prolonged latent phase b. Protracted active phase c. Arrest of active phase d. Protracted descent

ANS: C With an arrest of the active phase, the progress of labor has stopped. This client has not had any anticipated cervical change, thus indicating an arrest of labor. In the nulliparous woman a prolonged latent phase typically would last more than 20 hours. A protracted active phase, the first or second stage of labor, would be prolonged (slow dilation). With protracted descent, the fetus would fail to descend at an anticipated rate during the deceleration phase and second stage of labor. PTS: 1 DIF: Cognitive Level: Analysis REF: 455 OBJ: Nursing Process: Assessment, Diagnosis MSC: Client Needs: Health Promotion and Maintenance

A primigravida has just delivered a healthy infant girl. The nurse is about to administer erythromycin ointment in the infants eyes when the mother asks, What is that medicine for? The nurse responds: a. It is an eye ointment to help your baby see you better. b. It is to protect your baby from contracting herpes from your vaginal tract. c. Erythromycin is given prophylactically to prevent a gonorrheal infection. d. This medicine will protect your babys eyes from drying out over the next few days.

ANS: C With the prophylactic use of erythromycin, the incidence of gonococcal conjunctivitis has declined to less than 0.5%. Eye prophylaxis is administered at or shortly after birth to prevent ophthalmia neonatorum. Erythromycin has no bearing on enhancing vision, is used to prevent an infection caused by gonorrhea, not herpes, and is not used for eye lubrication. PTS: 1 DIF: Cognitive Level: Application REF: 670 OBJ: Nursing Process: Planning MSC: Client Needs: Health Promotion and Maintenance

A primigravida at 40 weeks of gestation is having uterine contractions every 1.5 to 2 minutes and says that they are very painful. Her cervix is dilated 2 cm and has not changed in 3 hours. The woman is crying and wants an epidural. What is the likely status of this womans labor? a. She is exhibiting hypotonic uterine dysfunction. b. She is experiencing a normal latent stage. c. She is exhibiting hypertonic uterine dysfunction. d. She is experiencing pelvic dystocia.

ANS: C Women who experience hypertonic uterine dysfunction, or primary dysfunctional labor, often are anxious firsttime mothers who are having painful and frequent contractions that are ineffective at causing cervical dilation or effacement to progress. With hypotonic uterine dysfunction, the woman initially makes normal progress into the active stage of labor; then the contractions become weak and inefficient or stop altogether. The contraction pattern seen in this woman signifies hypertonic uterine activity. Typically uterine activity in this phase occurs at 4- to 5-minute intervals lasting 30 to 45 seconds. Pelvic dystocia can occur whenever contractures of the pelvic diameters reduce the capacity of the bony pelvis, including the inlet, midpelvis, outlet, or any combination of these planes. PTS: 1 DIF: Cognitive Level: Application REF: 455 OBJ: Nursing Process: Diagnosis MSC: Client Needs: Health Promotion and Maintenance

The baseline fetal heart rate (FHR) is the average rate during a 10-minute segment. Changes in FHR are categorized as periodic or episodic. These patterns include both accelerations and decelerations. The labor nurse is evaluating the patients most recent 10-minute segment on the monitor strip and notes a late deceleration. This is likely to be caused by which physiologic alteration (Select all that apply)? a. Spontaneous fetal movement b. Compression of the fetal head c. Placental abruption d. Cord around the babys neck e. Maternal supine hypotension

ANS: C, E Late decelerations are almost always caused by uteroplacental insufficiency. Insufficiency is caused by uterine tachysystole, maternal hypotension, epidural or spinal anesthesia, IUGR, intraamniotic infection, or placental abruption. Spontaneous fetal movement, vaginal examination, fetal scalp stimulation, fetal reaction to external sounds, uterine contractions, fundal pressure and abdominal palpation are all likely to cause accelerations of the FHR. Early decelerations are most often the result of fetal head compression and may be caused by uterine contractions, fundal pressure, vaginal examination, and placement of an internal electrode. A variable deceleration is likely caused by umbilical cord compression. This may happen when the umbilical cord is around the babys neck, arm, leg, or other body part or when there is a short cord, a knot in the cord, or a prolapsed cord. PTS: 1 DIF: Cognitive Level: Analysis REF: 393 OBJ: Nursing Process: Assessment, Diagnosis MSC: Client Needs: Physiologic Integrity

The most common cause of decreased variability in the fetal heart rate (FHR) that lasts 30 minutes or less is: a. Altered cerebral blood flow. b. Fetal hypoxemia. c. Umbilical cord compression. d. Fetal sleep cycles.

ANS: D A temporary decrease in variability can occur when the fetus is in a sleep state. These sleep states do not usually last longer than 30 minutes. Altered fetal cerebral blood flow would result in early decelerations in the FHR. Fetal hypoxemia would be evidenced by tachycardia initially and then bradycardia. A persistent decrease or loss of FHR variability may be seen. Umbilical cord compression would result in variable decelerations in the FHR. PTS: 1 DIF: Cognitive Level: Comprehension REF: 390 OBJ: Nursing Process: Assessment MSC: Client Needs: Health Promotion and Maintenance

With regard to hemolytic diseases of the newborn, nurses should be aware that: a. Rh incompatibility matters only when an Rh-negative child is born to an Rh-positive mother. b. ABO incompatibility is more likely than Rh incompatibility to precipitate significant anemia. c. Exchange transfusions frequently are required in the treatment of hemolytic disorders. d. The indirect Coombs test is performed on the mother before birth; the direct Coombs test is performed on the cord blood after birth.

ANS: D An indirect Coombs test may be performed on the mother a few times during pregnancy. Only the Rh-positive child of an Rh-negative mother is at risk. ABO incompatibility is more common than Rh incompatibility but causes less severe problems; significant anemia, for instance, is rare with ABO. Exchange transfers are needed infrequently because of the decrease in the incidence of severe hemolytic disease in newborns from Rh incompatibility. PTS: 1 DIF: Cognitive Level: Comprehension REF: 680 OBJ: Nursing Process: Planning MSC: Client Needs: Health Promotion and Maintenance

A woman at 26 weeks of gestation is being assessed to determine whether she is experiencing preterm labor. What finding indicates that preterm labor is occurring? a. Estriol is not found in maternal saliva. b. Irregular, mild uterine contractions are occurring every 12 to 15 minutes. c. Fetal fibronectin is present in vaginal secretions. d. The cervix is effacing and dilated to 2 cm.

ANS: D Cervical changes such as shortened endocervical length, effacement, and dilation are predictors of imminent preterm labor. Changes in the cervix accompanied by regular contractions indicate labor at any gestation. Estriol is a form of estrogen produced by the fetus that is present in plasma at 9 weeks of gestation. Levels of salivary estriol have been shown to increase before preterm birth. Irregular, mild contractions that do not cause cervical change are not considered a threat. The presence of fetal fibronectin in vaginal secretions between 24 and 36 weeks of gestation could predict preterm labor, but it has only a 20% to 40% positive predictive value. Of more importance are other physiologic clues of preterm labor such as cervical changes. PTS: 1 DIF: Cognitive Level: Application REF: 461 OBJ: Nursing Process: Assessment, Planning MSC: Client Needs: Health Promotion and Maintenance

When providing an infant with a gavage feeding, which of the following should be documented each time? a. The infants abdominal circumference after the feeding b. The infants heart rate and respirations c. The infants suck and swallow coordination d. The infants response to the feeding

ANS: D Documentation of a gavage feeding should include the size of the feeding tube, the amount and quality of the residual from the previous feeding, the type and quantity of the fluid instilled, and the infants response to the procedure. Abdominal circumference is not measured after a gavage feeding. Vital signs may be obtained before feeding. However, the infants response to the feeding is more important. Some older infants may be learning to suck, but the important factor to document would be the infants response to the feeding (including attempts to suck). PTS: 1 DIF: Cognitive Level: Application REF: 700 OBJ: Nursing Process: Evaluation MSC: Client Needs: Physiologic Integrity

When assessing the relative advantages and disadvantages of internal and external electronic fetal monitoring, nurses comprehend that both: a. Can be used when membranes are intact. b. Measure the frequency, duration, and intensity of uterine contractions. c. May need to rely on the woman to indicate when uterine activity (UA) is occurring. d. Can be used during the antepartum and intrapartum periods.

ANS: D External monitoring can be used in both periods; internal monitoring can be used only in the intrapartum period. For internal monitoring the membranes must have ruptured, and the cervix must be sufficiently dilated. Internal monitoring measures the intensity of contractions; external monitoring cannot do this. With external monitoring, the woman may need to alert the nurse that UA is occurring; internal monitoring does not require this. PTS: 1 DIF: Cognitive Level: Comprehension REF: 385 OBJ: Nursing Process: Planning MSC: Client Needs: Health Promotion and Maintenance

Increasing the infusion rate of nonadditive intravenous fluids can increase fetal oxygenation primarily by: a. Maintaining normal maternal temperature. b. Preventing normal maternal hypoglycemia. c. Increasing the oxygen-carrying capacity of the maternal blood. d. Expanding maternal blood volume.

ANS: D Filling the mothers vascular system makes more blood available to perfuse the placenta and may correct hypotension. Increasing fluid volume may alter the maternal temperature only if she is dehydrated. Most intravenous fluids for laboring women are isotonic and do not provide extra glucose. Oxygen-carrying capacity is increased by adding more red blood cells. PTS: 1 DIF: Cognitive Level: Application REF: 396 OBJ: Nursing Process: Implementation MSC: Client Needs: Physiologic Integrity

With regard to the care management of preterm labor, nurses should be aware that: a. Because all women must be considered at risk for preterm labor and prediction is so hit-and-miss, teaching pregnant women the symptoms probably causes more harm through false alarms. b. Braxton Hicks contractions often signal the onset of preterm labor. c. Because preterm labor is likely to be the start of an extended labor, a woman with symptoms can wait several hours before contacting the primary caregiver. d. The diagnosis of preterm labor is based on gestational age, uterine activity, and progressive cervical change.

ANS: D Gestational age of 20 to 37 weeks, uterine contractions, and a cervix that is 80% effaced or dilated 2 cm indicates preterm labor. It is essential that nurses teach women how to detect the early symptoms of preterm labor. Braxton Hicks contractions resemble preterm labor contractions, but they are not true labor. Waiting too long to see a health care provider could result in not administering essential medications. Preterm labor is not necessarily long-term labor. PTS: 1 DIF: Cognitive Level: Comprehension REF: 445 OBJ: Nursing Process: Planning MSC: Client Needs: Safe and Effective Care Environment

What is one of the most common initial signs of nonreassuring fetal status? 1. Meconium-stained amniotic fluid 2. Cyanosis 3. Dehydration 4. Arrest of descent

Answer: 1 Explanation: 1. The most common initial signs of nonreassuring fetal status are meconium-stained amniotic fluid and changes in the fetal heart rate (FHR).

Nurses should be aware that the induction of labor: a. Can be achieved by external and internal version techniques. b. Is also known as a trial of labor (TOL). c. Is almost always done for medical reasons. d. Is rated for viability by a Bishop score.

ANS: D Induction of labor is likely to be more successful with a Bishop score of 9 or higher for first-time mothers and 5 or higher for veterans. Version is turning of the fetus to a better position by a physician for an easier or safer birth. A trial of labor is the observance of a woman and her fetus for several hours of active labor to assess the safety of vaginal birth. Two thirds of cases of induced labor are elective and are not done for medical reasons. PTS: 1 DIF: Cognitive Level: Comprehension REF: 461 OBJ: Nursing Process: Diagnosis MSC: Client Needs: Safe and Effective Care Environment

The standard of care for obstetrics dictates that an internal version may be used to manipulate the: a. Fetus from a breech to a cephalic presentation before labor begins. b. Fetus from a transverse lie to a longitudinal lie before cesarean birth. c. Second twin from an oblique lie to a transverse lie before labor begins. d. Second twin from a transverse lie to a breech presentation during vaginal birth.

ANS: D Internal version is used only during vaginal birth to manipulate the second twin into a presentation that allows it to be born vaginally. For internal version to occur, the cervix needs to be completely dilated. PTS: 1 DIF: Cognitive Level: Knowledge REF: 460 OBJ: Nursing Process: Assessment MSC: Client Needs: Physiologic Integrity

Which fetal heart rate (FHR) finding would concern the nurse during labor? a. Accelerations with fetal movement b. Early decelerations c. An average FHR of 126 beats/min d. Late decelerations

ANS: D Late decelerations are caused by uteroplacental insufficiency and are associated with fetal hypoxemia. They are considered ominous if persistent and uncorrected. Accelerations in the FHR are an indication of fetal well-being. Early decelerations in the FHR are associated with head compression as the fetus descends into the maternal pelvic outlet; they generally are not a concern during normal labor. PTS: 1 DIF: Cognitive Level: Analysis REF: 383 OBJ: Nursing Process: Assessment MSC: Client Needs: Health Promotion and Maintenance

When using intermittent auscultation (IA) for fetal heart rate, nurses should be aware that: a. They can be expected to cover only two or three clients when IA is the primary method of fetal assessment. b. The best course is to use the descriptive terms associated with electronic fetal monitoring (EFM) when documenting results. c. If the heartbeat cannot be found immediately, a shift must be made to EFM. d. Ultrasound can be used to find the fetal heartbeat and reassure the mother if initial difficulty was a factor.

ANS: D Locating fetal heartbeats often takes time. Mothers can be reassured verbally and by the ultrasound pictures if ultrasound is used to help locate the heartbeat. When used as the primary method of fetal assessment, auscultation requires a nurse-to-client ratio of one to one. Documentation should use only terms that can be numerically defined; the usual visual descriptions of EFM are inappropriate. PTS: 1 DIF: Cognitive Level: Comprehension REF: 384 OBJ: Nursing Process: Assessment, Planning MSC: Client Needs: Health Promotion and Maintenance

As a perinatal nurse you realize that a fetal heart rate that is tachycardic, is bradycardic, or has late decelerations or loss of variability is nonreassuring and is associated with: a. Hypotension. b. Cord compression. c. Maternal drug use. d. Hypoxemia.

ANS: D Nonreassuring heart rate patterns are associated with fetal hypoxemia. Fetal bradycardia may be associated with maternal hypotension. Fetal variable decelerations are associated with cord compression. Maternal drug use is associated with fetal tachycardia. PTS: 1 DIF: Cognitive Level: Analysis REF: 390 OBJ: Nursing Process: Assessment MSC: Client Needs: Health Promotion and Maintenance

What bacterial infection is definitely decreasing because of effective drug treatment? a. Escherichia coli infection b. Tuberculosis c. Candidiasis d. Group B streptococcal infection

ANS: D Penicillin has significantly decreased the incidence of group B streptococcal infection. E. coli may be increasing, perhaps because of the increasing use of ampicillin (resulting in a more virulent E. coli resistant to the drug). Tuberculosis is increasing in the United States and Canada. Candidiasis is a fairly benign fungal infection. PTS: 1 DIF: Cognitive Level: Comprehension REF: 674 OBJ: Nursing Process: Evaluation MSC: Client Needs: Health Promotion and Maintenance

Which assessment is least likely to be associated with a breech presentation? a. Meconium-stained amniotic fluid b. Fetal heart tones heard at or above the maternal umbilicus c. Preterm labor and birth d. Post-term gestation

ANS: D Post-term gestation is not likely to be seen with a breech presentation. The presence of meconium in a breech presentation may result from pressure on the fetal wall as it traverses the birth canal. Fetal heart tones heard at the level of the umbilical level of the mother are a typical finding in a breech presentation because the fetal back would be located in the upper abdominal area. Breech presentations often occur in preterm births. PTS: 1 DIF: Cognitive Level: Analysis REF: 457 OBJ: Nursing Process: Assessment MSC: Client Needs: Health Promotion and Maintenance

HIV may be perinatally transmitted: a. Only in the third trimester from the maternal circulation. b. From the use of unsterile instruments. c. Only through the ingestion of amniotic fluid. d. Through the ingestion of breast milk from an infected mother.

ANS: D Postnatal transmission of HIV through breastfeeding may occur. Transmission of HIV from the mother to the infant may occur transplacentally at various gestational ages. This is highly unlikely because most health care facilities must meet sterility standards for all instrumentation. Transmission of HIV may occur during birth from blood or secretions. PTS: 1 DIF: Cognitive Level: Comprehension REF: 676 OBJ: Nursing Process: Planning MSC: Client Needs: Physiologic Integrity: Reduction of Risk Potential

Human immunodeficiency virus (HIV) may be perinatally transmitted: a. Only in the third trimester from the maternal circulation. b. By a needlestick injury at birth from unsterile instruments. c. Only through the ingestion of amniotic fluid. d. Through the ingestion of breast milk from an infected mother.

ANS: D Postnatal transmission of HIV through breastfeeding may occur. Transmission of HIV from the mother to the infant may occur transplacentally at various gestational ages. Transmission close to or at the time of birth is thought to account for 50% to 80% of cases. PTS: 1 DIF: Cognitive Level: Comprehension REF: 676 OBJ: Nursing Process: Planning MSC: Client Needs: Physiologic Integrity

What is the most significant maternal risk factor for preterm birth? 1. Previous preterm birth 2. Smoking 3. Stress 4. Substance abuse

Answer: 1 Explanation: 1. The most significant maternal risk factor for preterm birth is a previous preterm birth.

For a woman at 42 weeks of gestation, which finding would require further assessment by the nurse? a. Fetal heart rate of 116 beats/min b. Cervix dilated 2 cm and 50% effaced c. Score of 8 on the biophysical profile d. One fetal movement noted in 1 hour of assessment by the mother

ANS: D Self-care in a post-term pregnancy should include performing daily fetal kick counts three times per day. The mother should feel four fetal movements per hour. If fewer than four movements have been felt by the mother, she should count for 1 more hour. Fewer than four movements in that hour warrants evaluation. Normal findings in a 42-week gestation include fetal heart rate of 116 beats/min, cervix dilated 20 cm and 50% effaced, and a score of 8 on the biophysical profile. PTS: 1 DIF: Cognitive Level: Application REF: 452 OBJ: Nursing Process: Assessment MSC: Client Needs: Health Promotion and Maintenance

While evaluating an external monitor tracing of a woman in active labor whose labor is being induced, the nurse notes that the fetal heart rate (FHR) begins to decelerate at the onset of several contractions and returns to baseline before each contraction ends. The nurse should: a. Change the womans position. b. Discontinue the oxytocin infusion. c. Insert an internal monitor. d. Document the finding in the clients record.

ANS: D The FHR indicates early decelerations, which are not an ominous sign and do not require any intervention. The nurse should simply document these findings. PTS: 1 DIF: Cognitive Level: Application REF: 399 OBJ: Nursing Process: Implementation MSC: Client Needs: Health Promotion and Maintenance

The priority nursing intervention after an amniotomy should be to: a. Assess the color of the amniotic fluid. b. Change the patients gown. c. Estimate the amount of amniotic fluid. d. Assess the fetal heart rate.

ANS: D The fetal heart rate must be assessed immediately after the rupture of the membranes to determine whether cord prolapse or compression has occurred. Secondary to FHR assessment, amniotic fluid amount, color, odor, and consistency is assessed. Dry clothing is important for patient comfort; however, it is not the top priority. PTS: 1 DIF: Cognitive Level: Application REF: 469 OBJ: Nursing Process: Implementation MSC: Client Needs: Physiologic Integrity

While completing a newborn assessment, the nurse should be aware that the most common birth injury is: a. To the soft tissues. b. Caused by forceps gripping the head on delivery. c. Fracture of the humerus and femur. d. Fracture of the clavicle.

ANS: D The most common birth injury is fracture of the clavicle (collarbone). It usually heals without treatment, although the arm and shoulder may be immobilized for comfort. PTS: 1 DIF: Cognitive Level: Knowledge REF: 665 OBJ: Nursing Process: Assessment MSC: Client Needs: Physiologic Integrity

Because of the premature infants decreased immune functioning, what nursing diagnosis should the nurse include in a plan of care for a premature infant? a. Delayed growth and development b. Ineffective thermoregulation c. Ineffective infant feeding pattern d. Risk for infection

ANS: D The nurse needs to understand that decreased immune functioning increases the risk for infection. Growth and development, thermoregulation, and feeding may be affected, although only indirectly. PTS: 1 DIF: Cognitive Level: Application REF: 684 OBJ: Nursing Process: Planning MSC: Client Needs: Physiologic Integrity

A pregnant woman at 37 weeks of gestation has had ruptured membranes for 26 hours. A cesarean section is performed for failure to progress. The fetal heart rate (FHR) before birth is 180 beats/min with limited variability. At birth the newborn has Apgar scores of 6 and 7 at 1 and 5 minutes and is noted to be pale and tachypneic. On the basis of the maternal history, the cause of this newborns distress is most likely to be: a. Hypoglycemia. b. Phrenic nerve injury. c. Respiratory distress syndrome. d. Sepsis.

ANS: D The prolonged rupture of membranes and the tachypnea (before and after birth) both suggest sepsis. An FHR of 180 beats/min is also indicative. This infant is at high risk for sepsis. PTS: 1 DIF: Cognitive Level: Comprehension REF: 666 OBJ: Nursing Process: Assessment MSC: Client Needs: Physiologic Integrity

The nurse practicing in a labor setting knows that the woman most at risk for uterine rupture is: a. A gravida 3 who has had two low-segment transverse cesarean births. b. A gravida 2 who had a low-segment vertical incision for delivery of a 10-pound infant. c. A gravida 5 who had two vaginal births and two cesarean births. d. A gravida 4 who has had all cesarean births.

ANS: D The risk of uterine rupture increases for the patient who has had multiple prior births with no vaginal births. As the number of prior uterine incisions increases, so does the risk for uterine rupture. Low-segment transverse cesarean scars do not predispose the patient to uterine rupture. PTS: 1 DIF: Cognitive Level: Comprehension REF: 479 OBJ: Nursing Process: Assessment MSC: Client Needs: Physiologic Integrity

A nurse may be called on to stimulate the fetal scalp: a. As part of fetal scalp blood sampling. b. In response to tocolysis. c. In preparation for fetal oxygen saturation monitoring. d. To elicit an acceleration in the fetal heart rate (FHR).

ANS: D The scalp can be stimulated using digital pressure during a vaginal examination. Fetal scalp blood sampling involves swabbing the scalp with disinfectant before a sample is collected. The nurse would stimulate the fetal scalp to elicit an acceleration of the FHR. Tocolysis is relaxation of the uterus. Fetal oxygen saturation monitoring involves the insertion of a sensor. PTS: 1 DIF: Cognitive Level: Application REF: 396 OBJ: Nursing Process: Implementation MSC: Client Needs: Health Promotion and Maintenance

In evaluating the effectiveness of magnesium sulfate for the treatment of preterm labor, what finding would alert the nurse to possible side effects? a. Urine output of 160 mL in 4 hours b. Deep tendon reflexes 2+ and no clonus c. Respiratory rate of 16 breaths/min d. Serum magnesium level of 10 mg/dL

ANS: D The therapeutic range for magnesium sulfate management is 5 to 8 mg/dL. A serum magnesium level of 10 mg/dL could lead to signs and symptoms of magnesium toxicity, including oliguria and respiratory distress. Urine output of 160 mL in 4 hours, deep tendon reflexes 2+ with no clonus, and respiratory rate of 16 breaths/min are normal findings. PTS: 1 DIF: Cognitive Level: Comprehension REF: 448 OBJ: Nursing Process: Evaluation MSC: Client Needs: Physiologic Integrity

A macrosomic infant is born after a difficult forceps-assisted delivery. After stabilization the infant is weighed, and the birth weight is 4550 g (9 pounds, 6 ounces). The nurses most appropriate action is to: a. Leave the infant in the room with the mother. b. Take the infant immediately to the nursery. c. Perform a gestational age assessment to determine whether the infant is large for gestational age. d. Monitor blood glucose levels frequently and observe closely for signs of hypoglycemia.

ANS: D This infant is macrosomic (more than 4000 g) and is at high risk for hypoglycemia. Blood glucose levels should be monitored frequently, and the infant should be observed closely for signs of hypoglycemia. Observation may occur in the nursery or in the mothers room, depending on the condition of the fetus. Regardless of gestational age, this infant is macrosomic. PTS: 1 DIF: Cognitive Level: Application REF: 668 OBJ: Nursing Process: Implementation MSC: Client Needs: Physiologic Integrity

Premature infants who exhibit 5 to 10 seconds of respiratory pauses followed by 10 to 15 seconds of compensatory rapid respiration are: a. Suffering from sleep or wakeful apnea. b. Experiencing severe swings in blood pressure. c. Trying to maintain a neutral thermal environment. d. Breathing in a respiratory pattern common to premature infants.

ANS: D This pattern is called periodic breathing and is common to premature infants. It may still require nursing intervention of oxygen and/or ventilation. Apnea is a cessation of respirations for 20 seconds or longer. It should not be confused with periodic breathing. PTS: 1 DIF: Cognitive Level: Comprehension REF: 684 OBJ: Nursing Process: Assessment MSC: Client Needs: Health Promotion and Maintenance

You are evaluating the fetal monitor tracing of your client, who is in active labor. Suddenly you see the fetal heart rate (FHR) drop from its baseline of 125 down to 80. You reposition the mother, provide oxygen, increase intravenous (IV) fluid, and perform a vaginal examination. The cervix has not changed. Five minutes have passed, and the fetal heart rate remains in the 80s. What additional nursing measures should you take? a. Scream for help. b. Insert a Foley catheter. c. Start Pitocin. d. Notify the care provider immediately.

ANS: D To relieve an FHR deceleration, the nurse can reposition the mother, increase IV fluid, and provide oxygen. If oxytocin is infusing, it should be discontinued. If the FHR does not resolve, the primary care provider should be notified immediately. Inserting a Foley catheter is an inappropriate nursing action. If the FHR were to continue in a nonreassuring pattern, a cesarean section could be warranted, which would require a Foley Catheter. However, the physician must make that determination. Pitocin may place additional stress on the fetus. PTS: 1 DIF: Cognitive Level: Evaluation REF: 395 OBJ: Nursing Process: Implementation MSC: Client Needs: Health Promotion and Maintenance

On day 3 of life, a newborn continues to require 100% oxygen by nasal cannula. The parents ask whether they can hold their infant during his next gavage feeding. Given that this newborn is physiologically stable, what response would the nurse give? a. Parents are not allowed to hold infants who depend on oxygen. b. You may hold only your babys hand during the feeding. c. Feedings cause more physiologic stress, so the baby must be closely monitored. Therefore, I dont think you should hold the baby. d. You may hold your baby during the feeding.

ANS: D You may hold your baby during the feeding is an accurate statement. Parental interaction via holding is encouraged during gavage feedings so that the infant will associate the feeding with positive interactions. Nasal cannula oxygen therapy allows for easier feedings and psychosocial interactions. The parent can swaddle the infant during gavage feedings to help the infant associate the feeding with positive interactions. Some parents like to do kangaroo care while gavage feeding their infant. Swaddling or kangaroo care during feedings provides positive interactions for the infant. PTS: 1 DIF: Cognitive Level: Application REF: 700 OBJ: Nursing Process: Planning MSC: Client Needs: Health Promotion and Maintenance

The home health nurse is admitting a client at 18 weeks who is pregnant with twins. Which nursing action is most important? 1. Teach the client about foods that are good sources of protein. 2. Assess the client's blood pressure in her upper right arm. 3. Determine whether the pregnancy is the result of infertility treatment. 4. Collect a cervicovaginal fetal fibronectin (fFN) specimen.

Answer: 1 Explanation: 1. A daily intake of 4000 kcal (minimum) and 135 g protein is recommended for a woman with normal-weight twins.

A woman is admitted to the birth setting in early labor. She is 3 cm dilated, -2 station, with intact membranes and FHR of 150 beats/min. Her membranes rupture spontaneously, and the FHR drops to 90 beats/min with variable decelerations. What would the initial response from the nurse be? 1. Perform a vaginal exam. 2. Notify the physician. 3. Place the client in a left lateral position. 4. Administer oxygen at 2 L per nasal cannula.

Answer: 1 Explanation: 1. A drop in fetal heart rate accompanied by variable decelerations is consistent with a prolapsed cord. The nurse would assess for prolapsed cord via vaginal examination.

The nurse is making client assignments for the next shift. Which client is most likely to experience a complicated labor pattern? 1. 34-year-old woman at 39 weeks' gestation with a large-for-gestational-age (LGA) fetus 2. 22-year-old woman at 23 weeks' gestation with ruptured membranes 3. 30-year-old woman at 41 weeks' gestation and estimated fetal weight 7 pounds 8 ounces 4. 43-year-old woman at 37 weeks' gestation with hypertension

Answer: 1 Explanation: 1. A risk factor for hypotonic uterine contraction patterns includes a large-for-gestational-age (LGA) fetus.

The nurse is teaching the parents of an infant with an inborn error of metabolism how to care for the infant at home. What information does teaching include? 1. Specially prepared formulas 2. Cataract problems 3. Low glucose concentrations 4. Administration of thyroid medication

Answer: 1 Explanation: 1. An afflicted PKU infant can be treated by a special diet that limits ingestion of phenylalanine. Special formulas low in phenylalanine, such as Lofenalac, Minafen, and Albumaid XP, are available.

A pregnant client is admitted to the emergency department with bleeding. The nurse realizes that the client might have placenta previa. Which signs would be suggestive of placenta previa? 1. Bright red vaginal bleeding 2. Sudden onset of vaginal bleeding 3. Firm and hard uterus 4. Change in the size of abdomen

Answer: 1 Explanation: 1. As the lower uterine segment contracts and dilates, the placental villi are torn from the uterine wall, causing bright red bleeding.

The nurse has received end-of-shift reports in the high-risk maternity unit. Which client should the nurse see first? 1. The client at 26 weeks' gestation with placenta previa experiencing blood on toilet tissue after a bowel movement 2. The client at 30 weeks' gestation with placenta previa whose fetal monitor strip shows late decelerations 3. The client at 35 weeks' gestation with grade I abruptio placentae in labor who has a strong urge to push 4. The client at 37 weeks' gestation with pregnancy-induced hypertension whose membranes ruptured spontaneously

Answer: 1 Explanation: 1. Assessment of the woman with placenta previa must be ongoing to prevent or treat complications that are potentially lethal to the mother and fetus. Painless, bright red vaginal bleeding is the best diagnostic sign of placenta previa. This client is the highest priority.

A 3-month-old baby who was born at 25 weeks has been exposed to prolonged oxygen therapy. Due to oxygen therapy, the nurse explains to the parents, their infant is at a greater risk for which of the following? 1. Visual impairment 2. Hyperthermia 3. Central cyanosis 4. Sensitive gag reflex

Answer: 1 Explanation: 1. Extremely premature newborns are particularly susceptible to injury of the delicate capillaries of the retina causing characteristic retinal changes known as retinopathy of prematurity (ROP). Judicious use of supplemental oxygen therapy in the premature infant has become the norm.

The nurse is caring for the newborn of a diabetic mother whose blood glucose level is 39 mg/dL. What should the nurse include in the plan of care for this newborn? 1. Offer early feedings with formula or breast milk. 2. Provide glucose water exclusively. 3. Evaluate blood glucose levels at 12 hours after birth. 4. Assess for hypothermia.

Answer: 1 Explanation: 1. IDMs whose serum glucose falls below 40 mg/dL should have early feedings with formula or breast milk (colostrum).

The client gave birth to a 7 pound, 14 ounce female 30 minutes ago. The placenta has not yet delivered. Manual removal of the placenta is planned. What should the nurse prepare to do? 1. Start an IV of lactated Ringer's. 2. Apply anti-embolism stockings. 3. Bottle-feed the infant. 4. Send the placenta to pathology.

Answer: 1 Explanation: 1. In women who do not have an epidural in place, intravenous sedation may be required because of the discomfort caused by the procedure. An IV is necessary.

Which assessment findings by the nurse would require obtaining a blood glucose level on the newborn? 1. Jitteriness 2. Sucking on fingers 3. Lusty cry 4. Axillary temperature of 98°F

Answer: 1 Explanation: 1. Jitteriness of the newborn is associated with hypoglycemia. Aggressive treatment is recommended after a single low blood glucose value if the infant shows this symptom.

The nurse is evaluating the effectiveness of phototherapy on a newborn. Which evaluation indicates a therapeutic response to phototherapy? 1. The newborn maintains a normal temperature 2. An increase of serum bilirubin levels 3. Weight loss 4. Skin blanching yellow

Answer: 1 Explanation: 1. Maintenance of temperature is an important aspect of phototherapy because the newborn is naked except for a diaper during phototherapy. The isolette helps the infant maintain his or her temperature while undressed.

The nurse is caring for the newborn of a diabetic mother. Which of the following should be included in the nurse's plan of care for this newborn? 1. Offer early feedings. 2. Administer an intravenous infusion of glucose. 3. Assess for hypercalcemia. 4. Assess for hyperbilirubinemia immediately after birth.

Answer: 1 Explanation: 1. Newborns of diabetic mothers may benefit from early feeding as they are extremely valuable in maintaining normal metabolism and lowering the possibility of such complications as hypoglycemia and hyperbilirubinemia.

In planning care for the fetal alcohol syndrome (FAS) newborn, which intervention would the nurse include? 1. Allow extra time with feedings. 2. Assign different personnel to the newborn each day. 3. Place the newborn in a well-lit room. 4. Monitor for hyperthermia.

Answer: 1 Explanation: 1. Newborns with fetal alcohol syndrome have feeding problems. Because of their feeding problems, these infants require extra time and patience during feedings.

The mother of a premature newborn questions why a gavage feeding catheter is placed in the mouth of the newborn and not in the nose. What is the nurse's best response? 1. "Most newborns are nose breathers." 2. "The tube will elicit the sucking reflex." 3. "A smaller catheter is preferred for feedings." 4. "Most newborns are mouth breathers."

Answer: 1 Explanation: 1. Orogastric insertion is preferable to nasogastric because most infants are obligatory nose breathers.

A woman has been having contractions since 4 a.m. At 8 a.m., her cervix is dilated to 5 cm. Contractions are frequent, and mild to moderate in intensity. Cephalopelvic disproportion (CPD) has been ruled out. After giving the mother some sedation so she can rest, what would the nurse anticipate preparing for? 1. Oxytocin induction of labor 2. Amnioinfusion 3. Increased intravenous infusion 4. Cesarean section

Answer: 1 Explanation: 1. Oxytocin is the drug of choice for labor augmentation or labor induction and may be administered as needed for hypotonic labor patterns.

A client is admitted to the birth setting in early labor. She is 3 cm dilated, -2 station, with intact membranes, and FHR of 150 bpm. Her membranes rupture spontaneously, and the FHR drops to 90 bpm with variable decelerations. What would the nurse's initial response be? 1. Perform a vaginal exam 2. Notify the physician 3. Place the client in a left lateral position 4. Administer oxygen at 2 L per nasal cannula

Answer: 1 Explanation: 1. Prolapsed umbilical cord can occur when the membranes rupture. The fetus is more likely to experience variable decelerations because the amniotic fluid is insufficient to keep pressure off the umbilical cord. A vaginal exam is the best way to confirm.

The parents of a newborn have just been told their infant has tetralogy of Fallot. The parents do not seem to understand the explanation given by the physician. What statement by the nurse is best? 1. "With this defect, not enough of the blood circulates through the lungs, leading to a lack of oxygen in the baby's body." 2. "The baby's aorta has a narrowing in a section near the heart that makes the left side of the heart work harder." 3. "The blood vessels that attach to the ventricles of the heart are positioned on the wrong sides of the heart." 4. "Your baby's heart doesn't circulate blood well because the left ventricle is smaller and thinner than normal."

Answer: 1 Explanation: 1. Tetralogy of Fallot is a cyanotic heart defect that comprises four abnormalities: pulmonary stenosis, ventricular septal defect, overriding aorta, and right ventricle hypertrophy. The severity of symptoms depends on the degree of pulmonary stenosis, the size of the ventricular septal defect, and the degree to which the aorta overrides the septal defect.

Two hours ago, a client at 39 weeks' gestation was 3 cm dilated, 40% effaced, and +1 station. Frequency of contractions was every 5 minutes with duration 40 seconds and intensity 50 mmHg. The current assessment is 4 cm dilated, 40% effaced, and +1 station. Frequency of contractions is now every 3 minutes with 40-50 seconds' duration and intensity of 40 mmHg. What would the priority intervention be? 1. Begin oxytocin after assessing for CPD. 2. Give terbutaline to stop the preterm labor. 3. Start oxygen at 8 L/min. 4. Have the anesthesiologist give the client an epidural.

Answer: 1 Explanation: 1. The client is having hypertonic contractions. Cephalopelvic disproportion (CPD) must be excluded. If CPD exists, oxytocin (Pitocin) augmentation should not be used. Oxytocin is the drug of choice for labor augmentation or labor induction.

The nurse prepares to admit to the nursery a newborn whose mother had meconium-stained amniotic fluid. The nurse knows this newborn might require which of the following? 1. Initial resuscitation 2. Vigorous stimulation at birth 3. Phototherapy immediately 4. An initial feeding of iron-enriched formula

Answer: 1 Explanation: 1. The presence of meconium in the amniotic fluid indicates that the fetus may be suffering from asphyxia. Meconium-stained newborns or newborns who have aspirated particulate meconium often have respiratory depression at birth and require resuscitation to establish adequate respiratory effort.

A nursing instructor is demonstrating how to perform a heel stick on a newborn. To obtain an accurate capillary hematocrit reading, what does the nursing instructor tell the student do? 1. Rub the heel vigorously with an isopropyl alcohol swab prior to obtaining blood. 2. Use a previous puncture site. 3. Cool the heel prior to obtaining blood. 4. Use a sterile needle and aspirate.

Answer: 1 Explanation: 1. The site should be cleaned by rubbing vigorously with 70% isopropyl alcohol swab. The friction produces local heat, which aids vasodilation.

The pregnant client at 41 weeks is scheduled for labor induction. She asks the nurse whether induction is really necessary. What response by the nurse is best? 1. "Babies can develop postmaturity syndrome, which increases their chances of having complications after birth." 2. "When infants are born 2 or more weeks after their due date, they have meconium in the amniotic fluid." 3. "Sometimes the placenta ages excessively, and we want to take care of that problem before it happens." 4. "The doctor wants to be proactive in preventing any problems with your baby if he gets any bigger."

Answer: 1 Explanation: 1. The term postmaturity applies to the infant who is born after 42 completed weeks of gestation and demonstrates characteristics of postmaturity syndrome.

The nurse is observing a student nurse care for a neonate undergoing intensive phototherapy. Which action by the student nurse indicates an understanding of how to provide this care? 1. Urine specific gravity is assessed each voiding. 2. Eye coverings are left off to help keep the baby calm. 3. Temperature is checked every 6 hours. 4. The infant is taken out of the isolette for diaper changes.

Answer: 1 Explanation: 1. This action is correct. Specific gravity provides one measure of urine concentration. Highly concentrated urine is associated with a dehydrated state. Weight loss is also a sign of developing dehydration in the newborn.

The nurse is caring for several pregnant clients. Which client should the nurse anticipate is most likely to have a newborn at risk for mortality or morbidity? 1. 37-year-old, with a history of multiple births and preterm deliveries who works in a chemical factory 2. 23-year-old of low socioeconomic status, unmarried 3. 16-year-old who began prenatal care at 30 weeks 4. 28-year-old with a history of gestational diabetes

Answer: 1 Explanation: 1. This client is at greatest risk because she has multiple risk factors: age over 35, high parity, history of preterm birth, and exposure to chemicals that might be toxic.

The nurse is admitting a client for a cerclage procedure. The client asks for information about the procedure. What is the nurse's most accurate response? 1. "A stitch is placed in the cervix to prevent a spontaneous abortion or premature birth." 2. "The procedure is done during the third trimester." 3. "Cerclage is always placed after the cervix has dilated and effaced." 4. "An uncomplicated elective cerclage may is done on inpatient basis."

Answer: 1 Explanation: 1. This is the correct description of cerclage.

The labor nurse is caring for a client at 38 weeks' gestation who has been diagnosed with symptomatic placenta previa. Which physician order should the nurse question? 1. Begin oxytocin drip rate at 0.5 milliunits/min. 2. Assess fetal heart rate every 10 minutes. 3. Weigh all vaginal pads. 4. Assess hematocrit and hemoglobin.

Answer: 1 Explanation: 1. This order should be questioned, as this client is not a good candidate for labor induction.

The nurse is caring for a 2-hour-old newborn whose mother is diabetic. The nurse assesses that the newborn is experiencing tremors. Which nursing action has the highest priority? 1. Obtain a blood calcium level. 2. Take the newborn's temperature. 3. Obtain a bilirubin level. 4. Place a pulse oximeter on the newborn.

Answer: 1 Explanation: 1. Tremors are a sign of hypocalcemia. Diabetic mothers tend to have decreased serum magnesium levels at term. This could cause secondary hypoparathyroidism in the infant.

During newborn resuscitation, how does the nurse evaluate the effectiveness of bag-and-mask ventilations? 1. The rise and fall of the chest 2. Sudden wakefulness 3. Urinary output 4. Adequate thermoregulation

Answer: 1 Explanation: 1. With proper resuscitation, chest movement is observed for proper ventilation. Pressure should be adequate to move the chest wall.

A 26-year-old client is having her initial prenatal appointment. The client reports to the nurse that she suffered a pelvic fracture in a car accident 3 years ago. The client asks whether her pelvic fracture might affect her ability to have a vaginal delivery. What response by the nurse is best? 1. "It depends on how your pelvis healed." 2. "You will need to have a cesarean birth." 3. "Please talk to your doctor about that." 4. "You will be able to delivery vaginally."

Answer: 1 Explanation: 1. Women with a history of pelvic fractures may also be at risk for cephalopelvic disproportion (CPD).

In caring for the premature newborn, the nurse must assess hydration status continually. Assessment parameters should include which of the following? Note: Credit will be given only if all correct choices and no incorrect choices are selected. Select all that apply. 1. Volume of urine output 2. Weight 3. Blood pH 4. Head circumference 5. Bowel sounds

Answer: 1, 2 Explanation: 1. In order to assess hydration status, volume of urine output must be evaluated. 2. In order to assess hydration status, the infant's weight must be evaluated.

The nurse is caring for a client who could be at risk for uterine rupture. The nurse is monitoring the fetus closely for which of the following? Note: Credit will be given only if all correct choices and no incorrect choices are selected. Select all that apply. 1. Late decelerations 2. Bradycardia 3. Loss of ability to determine fetal station 4. Tachycardia 5. Early decelerations

Answer: 1, 2, 3 Explanation: 1. Late decelerations could be seen with uterine rupture. 2. Bradycardia is seen if there is uterine rupture. 3. The uterus is not holding the fetus in place anymore if the uterus ruptures.

A client is admitted to the labor and delivery unit in active labor. What nursing diagnoses might apply to the client with suspected abruptio placentae? Note: Credit will be given only if all correct choices and no incorrect choices are selected. Select all that apply. 1. Fluid Volume, Deficient, Risk for, related to hypovolemia secondary to excessive blood loss 2. Tissue Perfusion: Peripheral, Ineffective, related to blood loss secondary to uterine atony following birth 3. Anxiety related to concern for own personal status and the baby's safety 4. Knowledge, Deficient related to lack of information about inherited genetic defects 5. Alteration in Respiratory Function related to blood loss

Answer: 1, 2, 3 Explanation: 1. Maternal and perinatal fetal mortality are concerns due to hypoxia. 2. Maternal and perinatal fetal mortality are concerns due to blood loss. 3. This mother would be anxious for herself and her baby.

Which findings would the nurse expect when assessing a newborn infected with syphilis? Note: Credit will be given only if all correct choices and no incorrect choices are selected. Select all that apply. 1. Rhinitis 2. Fissures on mouth corners 3. Red rash around anus 4. Lethargy 5. Large for gestational age

Answer: 1, 2, 3 Explanation: 1. Rhinitis is evident in the newborn exposed to syphilis. 2. Fissures on mouth corners and an excoriated upper lip indicate exposure to syphilis. 3. A red rash around the mouth and anus is observed.

The nurse caring for a client in labor anticipates fetal macrosomia and shoulder dystocia. Appropriate management of shoulder dystocia is essential in order to prevent which fetal complications? Note: Credit will be given only if all correct choices and no incorrect choices are selected. Select all that apply. 1. Brachial plexus injury 2. Fractured clavicle 3. Asphyxia 4. Neurological damage 5. Puerperal infection

Answer: 1, 2, 3, 4 Explanation: 1. Brachial plexus injury occurs due to improper or excessive traction applied to the fetal head. 2. Complications in macrosomia include fractured clavicles. 3. Complications in macrosomia include asphyxia of the fetus. 4. Neurological damage is a complication of macrosomia.

What fetal factors require a cesarean birth? (Select all) 1. Severe intrauterine growth restriction (IUGR) 2. Fetal anomalies 3. Unfavorable fetal position or presentation 4. Preterm birth 5. Lack of maternal attachment

Answer: 1, 2, 3, 4 Explanation: 1. Fetal factors such as severe intrauterine growth restriction (IUGR), preterm birth, fetal anomalies, nonreassuring fetal status, or unfavorable fetal position or presentation require cesarean birth. 2. Fetal factors such as severe intrauterine growth restriction (IUGR), preterm birth, fetal anomalies, nonreassuring fetal status, or unfavorable fetal position or presentation require cesarean birth. 3. Fetal factors such as severe intrauterine growth restriction (IUGR), preterm birth, fetal anomalies, nonreassuring fetal status, or unfavorable fetal position or presentation require cesarean birth. 4. Fetal factors such as severe intrauterine growth restriction (IUGR), preterm birth, fetal anomalies, nonreassuring fetal status, or unfavorable fetal position or presentation require cesarean birth.

The nurse is presenting a class on preterm labor, its causes, and treatments to a group of newly pregnant couples. Which statements regarding preterm labor are true? Note: Credit will be given only of all correct choices and no incorrect choices are selected. Select all that apply. 1. Antepartum hemorrhage can cause preterm labor. 2. Trauma can cause preterm labor. 3. Infection can cause preterm labor. 4. Magnesium sulfate is a drug used to stop contractions. 5. Sedatives and narcotics may be given to stop labor.

Answer: 1, 2, 3, 4 Explanation: 1. Hemorrhage from placenta previa or abruption can cause preterm labor. 2. Trauma to the abdomen or uterus can cause preterm labor. 3. Infections such as urinary tract infections can cause preterm labor. 4. Magnesium sulfate acts as a CNS depressant by decreasing the quantity of acetylcholine released by motor nerve impulses and thereby blocking neuromuscular transmission.

The nurse caring for a newborn with anemia would expect which initial laboratory data to be included in the initial assessment? Note: Credit will be given only if all correct choices and no incorrect choices are selected. Select all that apply. 1. Hemoglobin 2. Hematocrit 3. Reticulocyte count 4. Direct Coombs' test 5. Cord serum OgM

Answer: 1, 2, 3, 4 Explanation: 1. The initial laboratory workup for anemia should include hemoglobin measurements. 2. The initial laboratory workup for anemia should include hematocrit measurements. 3. The initial laboratory workup for anemia should include a reticulocyte count. 4. The direct Coombs' test reveals the presence of antibody-coated (sensitized) Rh-positive red blood cells in the newborn and should be included in the initial laboratory workup for anemia.

) In caring for a client with a uterine rupture, the nurse determines which nursing diagnoses to be appropriate? Note: Credit will be given only if all correct choices and no incorrect choices are selected. Select all that apply. 1. Gas Exchange, Impaired 2. Fear related to unknown outcome 3. Coping, Ineffective 4. Mobility: Physical, Impaired 5. Anxiety

Answer: 1, 2, 3, 5 Explanation: 1. Gas Exchange, Impaired diagnosis could apply to both mother and fetus. 2. The client would experience fear related to an unknown outcome. 3. Ineffective coping would be due to emergent situation secondary to uterine rupture. 5. There will be anxiety related to emergency procedures and unknown fetal outcome.

The nurse is presenting a class for nursing students on multiple-gestation pregnancy. Which statements about multiple-gestation pregnancies are accurate? Note: Credit will be given only if all correct choices and no incorrect choices are selected. Select all that apply. 1. Hypertension is a major maternal complication. 2. Gestational diabetes occurs more often. 3. Maternal anemia occurs. 4. Pulmonary embolism is 12 times more likely to develop during pregnancy with multiple gestations. 5. Multiple gestations are more likely to acquire HELLP.

Answer: 1, 2, 3, 5 Explanation: 1. Hypertension is a complication in multiple-gestation pregnancies. 2. Gestational diabetes occurs more often in multiple gestations. 3. Maternal anemia occurs because of demands of the multiple gestations. 5. Multiple gestations are more likely to acquire HELLP (hemolytic anemia, elevated liver enzymes, and low platelet count) syndrome, a complication resulting from eclampsia or preeclampsia.

Risk factors for tachysystole include which of the following? Note: Credit will be given only if all correct choices and no incorrect choices are selected. Select all that apply. 1. Cocaine use 2. Placental abruption 3. Low-dose oxytocin titration regimens 4. Uterine rupture 5. Smoking

Answer: 1, 2, 4 Explanation: 1. Cocaine use is a risk factor for tachysystole. 2. Placental abruption is a risk factor for tachysystole. 4. Uterine rupture is a risk factor for tachysystole.

When caring for a laboring client with oligohydramnios, what should the nurse be aware of? Note: Credit will be given only if all correct choices and no incorrect choices are selected. Select all that apply. 1. Increased risk of cord compression 2. Decreased variability 3. Labor progress is often more rapid than average 4. Presence of periodic decelerations 5. During gestation, fetal skin and skeletal abnormalities can occur

Answer: 1, 2, 4, 5 Explanation: 1. During the labor and birth, the lessened amounts of fluid reduce the cushioning effect for the umbilical cord, and cord compression is more likely to occur. 2. The nurse should evaluate the EFM tracing for the presence of nonperiodic decelerations or other nonreassuring signs (such as increasing or decreasing baseline, decreased variability, or presence of periodic decelerations). 4. The nurse should evaluate the EFM tracing for the presence of nonperiodic decelerations or other nonreassuring signs (such as increasing or decreasing baseline, decreased variability, or presence of periodic decelerations). 5. During the gestational period, fetal skin and skeletal abnormalities may occur because fetal movement is impaired as a result of inadequate amniotic fluid volume.

The nurse is caring for a newborn with full fontanelles and "setting sun" eyes. Which nursing interventions should be included in the care plan? Note: Credit will be given only if all correct choices and no incorrect choices are selected. Select all that apply. 1. Measure head circumference daily. 2. Assess for bulging fontanelles. 3. Avoid position changes. 4. Watch for signs of infection. 5. Use a gel pillow under the head.

Answer: 1, 2, 4, 5 Explanation: 1. The infant has congenital hydrocephalus. The nurse should measure and plot occipital-frontal baseline measurements, then measure head circumference once a day. 2. The infant has congenital hydrocephalus. Fontanelles should be checked for bulging and sutures for widening. 4. Infants with hydrocephalus are prone to infection. 5. The infant has congenital hydrocephalus. The enlarged head should be supported with a gel pillow.

The nurse is caring for a newborn with jaundice. The parents question why the newborn is not under phototherapy lights. The nurse explains that the fiber-optic blanket is beneficial because of which of the following? Note: Credit will be given only if all correct choices and no incorrect choices are selected. Select all that apply. 1. Lights can stay on all the time. 2. The eyes do not need to be covered. 3. The lights will need to be removed for feedings. 4. Newborns do not get overheated. 5. Weight loss is not a complication of this system.

Answer: 1, 2, 4, 5 Explanation: 1. With the fiber-optic blanket, the light stays on at all times. 2. The eyes do not have to be covered with a fiber optic blanket. 4. With the fiber-optic blanket, greater surface area is exposed and there are no thermoregulation issues. 5. Fluid and weight loss are not complications of fiber-optic blankets.

During labor, the client at 4 cm suddenly becomes short of breath, cyanotic, and hypoxic. The nurse must prepare or arrange immediately for which of the following? Note: Credit will be given only if all correct choices and no incorrect choices are selected. Select all that apply. 1. Intravenous access 2. Cesarean delivery 3. Immediate vaginal delivery 4. McRoberts maneuver 5. A crash cart

Answer: 1, 2, 5 Explanation: 1. When an amniotic fluid embolism is suspected, intravenous access is obtained as quickly as possible. 2. Shortness of breath, cyanosis, and hypoxia are symptoms of an amniotic fluid embolus, which necessitates immediate cesarean delivery. 5. The chances of a code are high, so the crash cart needs to be available.

What are the primary complications of placenta accrete? Note: Credit will be given only if all correct choices and no incorrect choices are selected. Select all that apply. 1. Maternal hemorrhage 2. Insomnia 3. Failure of the placenta to separate following birth of the infant 4. Autonomic dysreflexia 5. Shoulder dystocia

Answer: 1, 3 Explanation: 1. The primary complications of placenta accreta are maternal hemorrhage and failure of the placenta to separate following birth of the infant. 3. The primary complications of placenta accreta are maternal hemorrhage and failure of the placenta to separate following birth of the infant.

The nurse is teaching the parents of a newborn who has been exposed to HIV how to care for the newborn at home. Which instructions should the nurse emphasize? Note: Credit will be given only if all correct choices and no incorrect choices are selected. Select all that apply. 1. Use proper hand-washing technique. 2. Provide three feedings per day. 3. Place soiled diapers in a sealed plastic bag. 4. Cleanse the diaper changing area with a 1:10 bleach solution after each diaper change. 5. Take the temperature rectally.

Answer: 1, 3, 4 Explanation: 1. The nurse should instruct the parents on proper hand-washing technique. 3. The nurse should instruct parents to that soiled diapers are to be placed in plastic bags, sealed, and disposed of daily. 4. The nurse should instruct parents that the diaper-changing areas should be cleaned with a 1:10 dilution of household bleach after each diaper change.

The nurse is planning care for a client with hydramnios. For which interventions might the nurse need to prepare the client? Note: Credit will be given only if all correct choices and no incorrect choices are selected. Select all that apply. 1. Artificial rupture of the membranes 2. Amnioinfusion 3. Amniocentesis 4. Administration of prostaglandin synthesis inhibitor 5. Administration of indomethacin

Answer: 1, 3, 4, 5 Explanation: 1. Artificial rupture may be performed to remove the excessive fluid. 3. Amniocentesis may be performed to remove some excess fluid. 4. A prostaglandin synthesis inhibitor is used to treat hydramnios. 5. Indomethacin can decrease amniotic fluid by decreasing fetal urine output.

Which assessment findings would lead the nurse to suspect that a newborn might have a congenital heart defect? Note: Credit will be given only if all correct choices and no incorrect choices are selected. Select all that apply. 1. Cyanosis 2. Heart murmur 3. Bradycardia 4. Low urinary outputs 5. Tachypnea

Answer: 1, 3, 4, 5 Explanation: 1. Central cyanosis is defined as a visible, blue discoloration of the skin caused by decreased oxygen saturation levels and is a common manifestation of a cardiac defect. 3. The signs of congestive heart failure include tachycardia, not bradycardia. 4. The signs of congestive heart failure include low urinary output. 5. The signs of congestive heart failure include tachypnea.

The client has undergone an ultrasound, which estimated fetal weight at 4500 g (9 pounds 14 ounces). Which statement indicates that additional teaching is needed? 1. "Because my baby is big, I am at risk for excessive bleeding after delivery." 2. "Because my baby is big, his blood sugars could be high after he is born." 3. "Because my baby is big, my perineum could experience trauma during the birth." 4. "Because my baby is big, his shoulders could get stuck and a collarbone broken."

Answer: 2 Explanation: 2. Hypoglycemia, not hyperglycemia, is a potential complication experienced by a macrosomic fetus.

The nurse has admitted a woman with cervical insufficiency. The nurse is aware that causes of this condition include which of the following? Note: Credit will be given only if all correct choices and no incorrect choices are selected. Select all that apply. 1. Congenital factors 2. Intercourse during pregnancy 3. Infection 4. Increased uterine volume 5. Past cervical surgeries

Answer: 1, 3, 4, 5 Explanation: 1. Congenitally incompetent cervix may be found in women exposed to diethylstilbestrol (DES) or those with a bicornuate uterus. 3. Infection or trauma can cause acquired cervical incompetence. 4. Cervical insufficiency can occur in multiple-gestation pregnancies. 5. Previous elective abortion or cervical manipulation can lead to cervical insufficiency.

) The nurse is caring for a prenatal client. Reviewing the client's pregnancy history, the nurse identifies risk factors for an at-risk newborn, including which of the following? Note: Credit will be given only if all correct choices and no incorrect choices are selected. Select all that apply. 1. The mother's low socioeconomic status 2. Maternal age of 26 3. Mother's exposure to toxic chemicals 4. More than three previous deliveries 5. Maternal hypertension

Answer: 1, 3, 4, 5 Explanation: 1. Low socioeconomic status is associated with at-risk newborns. 3. Exposure to environmental dangers, such as toxic chemicals is associated with at-risk newborns. 4. Maternal factors such as multiparity are associated with at-risk newborns. 5. Preexisting maternal conditions, such as heart disease, diabetes, hypertension, hyperthyroidism, and renal disease are associated with at-risk newborns.

Which fetal/neonatal risk factors would lead the nurse to anticipate a potential need to resuscitate a newborn? Note: Credit will be given only if all correct choices and no incorrect choices are selected. Select all that apply. 1. Nonreassuring fetal heart rate pattern/sustained bradycardia 2. Fetal scalp/capillary blood sample pH greater than 7.25 3. History of meconium in amniotic fluid 4. Prematurity 5. Significant intrapartum bleeding

Answer: 1, 3, 4, 5 Explanation: 1. Nonreassuring fetal heart rate pattern/sustained bradycardia would be considered a potential need to resuscitate a newborn. 3. History of meconium in amniotic fluid would be considered a potential need to resuscitate a newborn. 4. Prematurity would be considered a potential need to resuscitate a newborn. 5. Significant intrapartum bleeding would be considered a potential need to resuscitate a newborn.

Antibiotics have been ordered for a newborn with an infection. Which interventions would the nurse prepare to implement? Note: Credit will be given only if all correct choices and no incorrect choices are selected. Select all that apply. 1. Obtain skin cultures. 2. Restrict parental visits. 3. Evaluate bilirubin levels. 4. Administer oxygen as ordered. 5. Observe for signs of hypoglycemia.

Answer: 1, 3, 4, 5 Explanation: 1. The nurse will assist in obtaining skin cultures. Skin cultures are taken of any lesions or drainage from lesions or reddened areas. 3. The nurse will observe for hyperbilirubinemia, anemia, and hemorrhagic symptoms. 4. The nurse will administer oxygen as ordered. 5. The nurse will observe for signs of hypoglycemia.

When planning care for the premature newborn diagnosed with respiratory distress syndrome, which potential complications would the nurse anticipate? Note: Credit will be given only if all correct choices and no incorrect choices are selected. Select all that apply. 1. Hypoxia 2. Respiratory alkalosis 3. Metabolic acidosis 4. Massive atelectasis 5. Pulmonary edema

Answer: 1, 3, 4, 5 Explanation: 1. The physiologic alterations of RDS can produce hypoxia as a complication. As a result of hypoxia, the pulmonary vasculature constricts, pulmonary vascular resistance increases, and pulmonary blood flow is reduced. 3. The physiologic alterations of RDS can produce metabolic acidosis as a complication. Because cells lack oxygen, the newborn begins an anaerobic pathway of metabolism, with an increase in lactate levels and a resulting base deficit. 4. The physiologic alterations of RDS can produce massive atelectasis as a complication. Upon expiration, the instability increases the atelectasis, which causes hypoxia and acidosis because of the lack of gas exchange. 5. The physiologic alterations of RDS can produce pulmonary edema as a complication. Opacification of the lungs on X-ray image may be due to massive atelectasis, diffuse alveolar infiltrate, or pulmonary edema.

The nurse is caring for an infant of a diabetic mother. Which potential complications would the nurse consider in planning care for this newborn? Note: Credit will be given if all correct choices and no incorrect choices are selected. Select all that apply. 1. Tremors 2. Hyperglycemia 3. Hyperbilirubinemia 4. Respiratory distress syndrome 5. Birth trauma

Answer: 1, 3, 4, 5 Explanation: 1. Tremors are a clinical sign of hypocalcemia. 3. Hyperbilirubinemia is caused by slightly decreased extracellular fluid volume, which increases the hematocrit level. 4. Respiratory distress syndrome (RDS) is a complication that occurs more frequently in newborns of diabetic mothers whose diabetes is not well controlled. 5. Because most IDMs are macrosomic, trauma may occur during labor and vaginal birth resulting in shoulder dystocia, brachial plexus injuries, subdural hemorrhage, cephalohematoma, and asphyxia.

The nurse is caring for the newborn of a drug-addicted mother. Which assessment findings would be typical for this newborn? Note: Credit will be given only if all correct choices and no incorrect choices are selected. Select all that apply. 1. Hyperirritability 2. Decreased muscle tone 3. Exaggerated reflexes 4. Low pitched cry 5. Transient tachypnea

Answer: 1, 3, 5 Explanation: 1. Newborns born to drug-addicted mothers exhibit hyperirritability. 3. Newborns born to drug-addicted mothers exhibit exaggerated reflexes. 5. Newborns born to drug-addicted mothers exhibit transient tachypnea.

The nurse is admitting a client who was diagnosed with hydramnios. The client asks why she has developed this condition. The nurse should explain that hydramnios is sometimes associated with which of the following? Note: Credit will be given only if all correct choices and no incorrect choices are selected. Select all that apply. 1. Rh sensitization 2. Postmaturity syndrome 3. Renal malformation or dysfunction 4. Maternal diabetes 5. Large-for-gestational-age infants

Answer: 1, 4 Explanation: 1. Hydramnios is associated with Rh sensitization. 4. Hydramnios is associated with maternal diabetes.

If oligohydramnios occurs in the first part of pregnancy, the nurse knows that there is a danger of which of the following? 1. Major congenital anomalies 2. Fetal adhesions 3. Maternal diabetes 4. Rh sensitization

Answer: 2 Explanation: 2. If oligohydramnios occurs in the first part of pregnancy, there is a danger of fetal adhesions (one part of the fetus may adhere to another part).

A client at 32 weeks' gestation is admitted with painless vaginal bleeding. Placenta previa has been confirmed by ultrasound. What should be included in the nursing plan? Note: Credit will be given only if all correct choices and no incorrect choices are selected. Select all that apply. 1. No vaginal exams 2. Encouraging activity 3. No intravenous access until labor begins 4. Evaluating fetal heart rate with an external monitor 5. Monitoring blood loss, pain, and uterine contractility

Answer: 1, 4, 5 Explanation: 1. Expectant management of placenta previa is made by localizing the placenta via tests that require no vaginal examination. 4. Expectant management of placenta previa, when the client is at less than 37 weeks' gestation, includes evaluating FHR with an external monitor. 5. Expectant management of placenta previa, when the client is at less than 37 weeks' gestation, includes monitoring blood loss, pain, and uterine contractility.

Many newborns exposed to HIV/AIDS show signs and symptoms of disease within days of birth that include which of the following? Note: Credit will be given only if all correct choices and no incorrect choices are selected. Select all that apply. 1. Swollen glands 2. Hard stools 3. Smaller than average spleen and liver 4. Rhinorrhea 5. Interstitial pneumonia

Answer: 1, 4, 5 Explanation: 1. Signs that may be seen in the early infancy period include swollen glands. 4. Signs that may be seen in the early infancy period include rhinorrhea. 5. Signs that may be seen in the early infancy period include interstitial pneumonia.

The nurse is preparing an educational in-service presentation about jaundice in the newborn. What content should the nurse include in this presentation? Note: Credit will be given only if all correct choices and no incorrect choices are selected. Select all that apply. 1. Physiologic jaundice occurs after 24 hours of age. 2. Pathologic jaundice occurs after 24 hours of age. 3. Phototherapy increases serum bilirubin levels. 4. The need for phototherapy depends on the bilirubin level and age of the infant. 5. Kernicterus causes irreversible neurological damage.

Answer: 1, 5 Explanation: 1. Physiologic or neonatal jaundice is a normal process that occurs during transition from intrauterine to extrauterine life and appears after 24 hours of life. 5. Kernicterus refers to the deposition of unconjugated bilirubin in the basal ganglia of the brain and to permanent neurologic sequelae of untreated hyperbilirubinemia.

Slowly removing some amniotic fluid is a treatment for hydramnios. What consequence can occur with the withdrawal of fluid? 1. Preterm labor 2. Prolapsed cord 3. Preeclampsia 4. Placenta previa

Answer: 2 Explanation: 2. A needle or a fetal scalp electrode is used to make a small puncture in the amniotic sac. There is a risk that the force of the fluid could make a larger hole in the amniotic sac, thus increasing the risk of a prolapsed cord.

The special care nursery nurse is working with parents of a 3-day-old infant who was born with myelomeningocele and has developed an infection. Which statement from the mother is unexpected? 1. "If I had taken better care of myself, this wouldn't have happened." 2. "I've been sleeping very well since I had the baby." 3. "This is probably the doctor's fault." 4. "If I hadn't seen our baby's birth, I wouldn't believe she is ours."

Answer: 2 Explanation: 2. A sick infant is a source of great anxiety for parents. This response is from the mother would be unexpected.

One day after giving birth vaginally, a client develops painful vesicular lesions on her perineum and vulva. She is diagnosed with a primary herpes simplex 2 infection. What is the expected care for her neonate? 1. Meticulous hand washing and antibiotic eye ointment administration. 2. Intravenous acyclovir (Zovirax) and contact precautions. 3. Cultures of blood and CSF and serial chest x-rays every 12 hours. 4. Parental rooming-in and four intramuscular injections of penicillin.

Answer: 2 Explanation: 2. Administering intravenous acyclovir (Zovirax) and contact precautions are appropriate measures for an infant at risk for developing herpes simplex 2 infection.

During labor, the fetus was in a brow presentation, but after a prolonged labor, the fetus converted to face presentation and was delivered vaginally with forceps assist. What should the nurse explain to the parents? 1. The infant will need to be observed for meconium aspiration. 2. Facial edema and head molding will subside in a few days. 3. The infant will be given prophylactic antibiotics. 4. Breastfeeding will need to be delayed for a day or two.

Answer: 2 Explanation: 2. Any facial edema and head molding that result from the use of forceps at birth will subside in a few days.

What is required for any women receiving oxytocin (Pitocin)? 1. CPR 2. Continuous electronic fetal monitoring 3. Administering oxygen by mask 4. Nonstress test

Answer: 2 Explanation: 2. Continuous electronic fetal monitoring (EFM) is required for any women receiving oxytocin (Pitocin).

What can be determined based on ultrasound visualization or the lack of visualization of an intertwin membrane? 1. Toxicity 2. Amnionicity 3. Variability 4. Prematurity

Answer: 2 Explanation: 2. Evidence supports the use of ultrasound for accurately determining chorionicity and amnionicity in multiple pregnancies. Determination of amnionicity is based on ultrasound visualization or the lack of visualization of an intertwin membrane.

A mother who is HIV-positive has given birth to a term female. What plan of care is most appropriate for this infant? 1. Test with a HIV serologic test at 8 months. 2. Begin prophylactic AZT (Zidovudine) administration. 3. Provide 4 to 5 large feedings throughout the day. 4. Encourage the mother to breastfeed the child.

Answer: 2 Explanation: 2. For infants, AZT is started prophylactically 2 mg/kg/dose PO every 6 hours beginning as soon after birth as possible and continuing for 6 weeks.

The nurse is admitting a client with possible hydramnios. When is hydramnios most likely suspected? 1. Hydramnios is most likely suspected when there is less amniotic fluid than normal for gestation. 2. Hydramnios is most likely suspected when the fundal height increases disproportionately to the gestation. 3. Hydramnios is most likely suspected when the woman has a twin gestation. 4. Hydramnios is most likely suspected when the quadruple screen comes back positive.

Answer: 2 Explanation: 2. Hydramnios should be suspected when the fundal height increases out of proportion to the gestational age.

A woman has been in labor for 16 hours. Her cervix is dilated to 3 cm and is 80% effaced. The fetal presenting part is not engaged. The nurse would suspect which of the following? 1. Breech malpresentation 2. Fetal demise 3. Cephalopelvic disproportion (CPD) 4. Abruptio placentae

Answer: 3 Explanation: 3. Cephalopelvic disproportion (CPD) prevents the presenting part from becoming engaged.

A client in labor is found to have meconium-stained amniotic fluid upon rupture of membranes. At delivery, the nurse finds the infant to have depressed respirations and a heart rate of 80. What does the nurse anticipate? 1. Delivery of the neonate on its side with head up, to facilitate drainage of secretions. 2. Direct tracheal suctioning by specially trained personnel. 3. Preparation for the immediate use of positive pressure to expand the lungs. 4. Suctioning of the oropharynx when the newborn's head is delivered.

Answer: 2 Explanation: 2. If the infant has absent or depressed respirations, heart rate less than 100 beats/min, or poor muscle tone, direct tracheal suctioning by specially trained personnel is recommended.

The nurse is planning care for four infants who were born on this shift. The infant who will require the most detailed assessment is the one whose mother has which of the following? 1. A history of obsessive-compulsive disorder (OCD) 2. Chlamydia 3. Delivered six other children by cesarean section 4. A urinary tract infection (UTI)

Answer: 2 Explanation: 2. Infants born to mothers with chlamydia infections are at risk for neonatal pneumonia and conjunctivitis, and require close observation of the respiratory status and eyes. Page

Mild or chronic anemia in an infant may be treated adequately which of the following? 1. Transfusions with O-negative or typed and cross-matched packed red cells 2. Iron supplements or iron-fortified formulas 3. Steroid therapy 4. Antibiotics or antivirals

Answer: 2 Explanation: 2. Mild or chronic anemia in an infant may be treated adequately with iron supplements or iron-fortified formulas.

The nurse notes that a 36-hour-old newborn's serum bilirubin level has increased from 14 mg/dL to 16.6 mg/dL in an 8-hour period. What nursing intervention would be included in the plan of care for this newborn? 1. Continue to observe 2. Begin phototherapy 3. Begin blood exchange transfusion 4. Stop breastfeeding

Answer: 2 Explanation: 2. Neonatal hyperbilirubinemia must be considered pathologic if the serum bilirubin concentration is rising by more than 0.2 mg/dL per hour. If the newborn is over 24 hours old, which is past the time where an increase in bilirubin would result from pathologic causes, phototherapy may be the treatment of choice to prevent the possible complications of kernicterus.

The nurse is caring for a client with hydramnios. What will the nurse watch for? 1. Possible intrauterine growth restriction 2. Newborn congenital anomalies 3. Newborn postmaturity and renal malformations 4. Fetal adhesions

Answer: 2 Explanation: 2. Newborn congenital anomalies occur with hydramnios

The nurse should anticipate the labor pattern for a fetal occiput posterior position to be which of the following? 1. Shorter than average during the latent phase 2. Prolonged as regards the overall length of labor 3. Rapid during transition 4. Precipitous

Answer: 2 Explanation: 2. Occiput posterior (OP) position of the fetus is the most common fetal malposition and occurs when the head remains in the direct OP position throughout labor. This can prolong the overall length of labor.

The nurse is preparing an educational session on phenylketonuria for a family whose neonate has been diagnosed with the condition. Which statement by a parent indicates that teaching was effective? 1. "This condition occurs more frequently among Japanese people." 2. "We must be very careful to avoid most proteins to prevent brain damage." 3. "Carbohydrates can cause our baby to develop cataracts and liver damage." 4. "Our baby's thyroid gland isn't functioning properly."

Answer: 2 Explanation: 2. PKU is the inability to metabolize phenylalanine, an amino acid found in most dietary protein sources. Excessive accumulation of phenylalanine and its abnormal metabolites in the brain tissue leads to progressive, irreversible intellectual disability.

The client delivered 30 minutes ago. Her blood pressure and pulse are stable. Vaginal bleeding is scant. The nurse should prepare for which procedure? 1. Abdominal hysterectomy 2. Manual removal of the placenta 3. Repair of perineal lacerations 4. Foley catheterization

Answer: 2 Explanation: 2. Retention of the placenta beyond 30 minutes after birth is termed retained placenta. Manual removal of the placenta is then performed.

A client was admitted to the labor area at 5 cm with ruptured membranes about 14 hours ago. What assessment data would be most beneficial for the nurse to collect? 1. Blood pressure 2. Temperature 3. Pulse 4. Respiration

Answer: 2 Explanation: 2. Rupture of membranes places the mother at risk for infection. The temperature is the primary and often the first indication of a problem.

The nurse is caring for a jaundiced infant receiving bank light phototherapy in an isolette. Which finding requires an immediate intervention? 1. Eyes are covered, no clothing on, diaper in place 2. Axillary temperature 99.7°F 3. Infant removed from the isolette for breastfeeding 4. Loose bowel movement

Answer: 2 Explanation: 2. Temperature assessment is indicated to detect hypothermia or hyperthermia. Normal temperature ranges are 97.7°F-98.6°F. Vital signs should be monitored every 4 hours with axillary temperatures.

What indications would lead the nurse to suspect sepsis in a newborn? 1. Respiratory distress syndrome developing 48 hours after birth 2. Temperature of 97.0°F 2 hours after warming the infant from 97.4°F 3. Irritability and flushing of the skin at 8 hours of age 4. Bradycardia and tachypnea developing when the infant is 36 hours old

Answer: 2 Explanation: 2. Temperature instability is often seen with sepsis. Fever is rare in a newborn.

In succenturiate placenta, one or more accessory lobes of fetal villi have developed on the placenta, with vascular connections of fetal origin. What is the gravest maternal danger? 1. Cord prolapse 2. Postpartum hemorrhage 3. Paroxysmal hypertension 4. Brachial plexus injury

Answer: 2 Explanation: 2. The gravest maternal danger is postpartum hemorrhage if this minor lobe is severed from the placenta and remains in the uterus.

The client at 38 weeks' gestation has been diagnosed with oligohydramnios. Which statement indicates that teaching about the condition has been effective? 1. "My gestational diabetes might have caused this problem to develop." 2. "When I go into labor, I should come to the hospital right away." 3. "This problem was diagnosed with blood and urine tests." 4. "Women with this condition usually do not have a cesarean birth."

Answer: 2 Explanation: 2. The incidence of cord compression and resulting fetal distress is high when there is an inadequate amount of amniotic fluid. The client with oligohydramnios should come to the hospital in early labor.

The nurse is assessing a 2-hour-old newborn delivered by cesarean at 38 weeks. The amniotic fluid was clear. The mother had preeclampsia. The newborn has a respiratory rate of 80, is grunting, and has nasal flaring. What is the most likely cause of this infant's condition? 1. Meconium aspiration syndrome 2. Transient tachypnea of the newborn 3. Respiratory distress syndrome 4. Prematurity of the neonate

Answer: 2 Explanation: 2. The infant is term and was born by cesarean, and is most likely experiencing transient tachypnea of the newborn.

A fetal weight is estimated at 4490 grams in a client at 38 weeks' gestation. Counseling should occur before labor regarding which of the following? 1. Mother's undiagnosed diabetes 2. Likelihood of a cesarean delivery 3. Effectiveness of epidural anesthesia with a large fetus 4. Need for early delivery

Answer: 2 Explanation: 2. The likelihood of a cesarean delivery with a fetus over 4000 grams is high. This should be discussed with the client before labor.

Dystocia encompasses many problems in labor. What is the most common? 1. Meconium-stained amniotic fluid 2. Dysfunctional uterine contractions 3. Cessation of contractions 4. Changes in the fetal heart rate

Answer: 2 Explanation: 2. The most common problem is dysfunctional (or uncoordinated) uterine contractions that result in a prolongation of labor.

An HIV-positive mother delivered 2 days ago. The infant will be placed in foster care. The nurse is planning discharge teaching for the foster parents on how to care for the newborn at home. Which instructions should the nurse include? 1. Do not add food supplements to the baby's diet. 2. Place soiled diapers in a sealed plastic bag. 3. Wash soiled linens in cool water with bleach. 4. Shield the baby's eyes from bright lights.

Answer: 2 Explanation: 2. The nurse should instruct the parents about proper hand-washing techniques, about proper disposal of soiled diapers, and to wear gloves when diapering.

During the nursing assessment of a woman with ruptured membranes, the nurse suspects a prolapsed umbilical cord. What would the nurse's priority action be? 1. To help the fetal head descend faster 2. To use gravity and manipulation to relieve compression on the cord 3. To facilitate dilation of the cervix with prostaglandin gel 4. To prevent head compression

Answer: 2 Explanation: 2. The top priority is to relieve compression on the umbilical cord to allow blood flow to reach the fetus. It is because some obstetric maneuvers to relieve cord compression are complicated that cesarean birth is sometimes necessary.

The client with blood type O Rh-negative has given birth to an infant with blood type O Rh-positive. The infant has become visibly jaundiced at 12 hours of age. The mother asks why this is happening. What is the best response by the nurse? 1. "The RhoGAM you received at 28 weeks' gestation did not prevent alloimmunization." 2. "Your body has made antibodies against the baby's blood that are destroying her red blood cells." 3. "The red blood cells of your baby are breaking down because you both have type O blood." 4. "Your baby's liver is too immature to eliminate the red blood cells that are no longer needed."

Answer: 2 Explanation: 2. This explanation is accurate and easy for the client to understand. Newborns of Rh-negative and O blood type mothers are carefully assessed for blood type status, appearance of jaundice, and levels of serum bilirubin.

The nurse is planning an in-service educational program to talk about disseminated intravascular coagulation (DIC). The nurse should identify which conditions as risk factors for developing DIC? Note: Credit will be given only if all correct choices and no incorrect choices are selected. Select all that apply. 1. Diabetes mellitus 2. Abruptio placentae 3. Fetal demise 4. Multiparity 5. Preterm labor

Answer: 2, 3 Explanation: 2. As a result of the damage to the uterine wall and the retroplacental clotting with covert abruption, large amounts of thromboplastin are released into the maternal blood supply, which in turn triggers the development of disseminated intravascular coagulation (DIC) and the resultant hypofibrinogenemia. 3. Perinatal mortality associated with abruptio placentae is approximately 25%. If fetal hypoxia progresses unchecked, irreversible brain damage or fetal demise may result.

The nurse is assessing a newborn diagnosed with physiologic jaundice. Which findings would the nurse expect? Note: Credit will be given only if all correct choices and no incorrect choices are selected. Select all that apply. 1. Jaundice present within the first 24 hours of life 2. Appearance of jaundice symptoms after 24 hours of life 3. Yellowish coloration of the sclera of the eyes 4. Cephalohematoma or excessive bruising 5. Cyanosis

Answer: 2, 3 Explanation: 2. Physiologic or neonatal jaundice is a normal process that occurs during transition from intrauterine to extrauterine life and appears after 24 hours of life. 3. Jaundice is a yellowish coloration of the skin and sclera of the eyes that develops from the deposit of yellow pigment bilirubin in lipid/fat-containing tissues.

Nonreassuring fetal status often occurs with a tachysystole contraction pattern. Intrauterine resuscitation measures may become warranted and can include which of the following measures? Note: Credit will be given only if all correct choices and no incorrect choices are selected. Select all that apply. 1. Position the woman on her right side. 2. Apply oxygen via face mask. 3. Call for anesthesia provider for support. 4. Increase intravenous fluids by at least 700 mL bolus. 5. Call the physician/CNM to the bedside.

Answer: 2, 3, 4 Explanation: 2. The nurse would apply oxygen via face mask. 3. The nurse would call for anesthesia provider for support. 4. The nurse would increase intravenous fluids by at least 500 mL bolus.

The nurse knows that the maternal risks associated with postterm pregnancy include which of the following? Note: Credit will be given only if all correct choices and no incorrect choices are selected. Select all that apply. 1. Polyhydramnios 2. Maternal hemorrhage 3. Maternal anxiety 4. Forceps-assisted delivery 5. Perineal damage

Answer: 2, 3, 4, 5 Explanation: 2. Maternal symptoms and complications in postterm pregnancy may include maternal hemorrhage. 3. Maternal symptoms and complications in postterm pregnancy may include maternal anxiety. 4. Maternal symptoms and complications in postterm pregnancy may include an operative vaginal birth with forceps or vacuum extractor. 5. Maternal symptoms and complications in postterm pregnancy may include perineal trauma and damage.

What would be a normal cervical dilatation rate in a first-time mother ("primip")? 1. 1.5 cm per hour 2. Less than 1 cm cervical dilatation per hour 3. 1 cm per hour 4. Less than 0.5 cm per hour

Answer: 3 Explanation: 3. Dilatation in a "multip" is about 1.5 cm per hour. Dilation in a "primip" is 1 cm per hour.

Which of the following potential problems would the nurse consider when planning care for a client with a persistent occiput posterior position of the fetus? Note: Credit will be given only if all correct choices and no incorrect choices are selected. Select all that apply. 1. Increased fetal mortality 2. Severe perineal lacerations 3. Ceasing of labor progress 4. Fetus born in posterior position 5. Intense back pain during labor

Answer: 2, 3, 4, 5 Explanation: 2. The woman can have third- or fourth-degree perineal laceration or extension of a midline episiotomy. 3. Sometimes labor progress ceases if the fetus fails to rotate to an occiput anterior position. 4. Occiput posterior positions are associated with a higher incidence of vacuum-assisted births. 5. The woman usually experiences intense back pain in the small of her back throughout labor.

Which nursing interventions are appropriate when caring for the newborn undergoing phototherapy? Note: Credit will be given only if all correct choices and no incorrect choices are selected. Select all that apply. 1. Cover the newborn's eyes at all times, even when not under the lights. 2. Close the newborn's eyelids before applying eye patches. 3. Inspect the eyes each shift for conjunctivitis. 4. Keep the baby swaddled in a blanket to prevent heat loss. 5. Reposition the baby every 2 hours.

Answer: 2, 3, 5 Explanation: 2. Apply eye patches over the newborn's closed eyes during exposure to banks of phototherapy. 3. Discontinue conventional phototherapy and remove the eye patches at least once per shift to assess the eyes for the presence of conjunctivitis. 5. Repositioning allows equal exposure of all skin areas and prevents pressure areas.

A NICU nurse plans care for a preterm newborn that will provide opportunities for development. Which interventions support development in a preterm newborn in a NICU? Note: Credit will be given only if all correct choices and no incorrect choices are selected. Select all that apply. 1. Schedule care throughout the day. 2. Silence alarms quickly. 3. Place a blanket over the top portion of the incubator. 4. Do not offer a pacifier. 5. Dim the lights.

Answer: 2, 3, 5 Explanation: 2. Noise levels can be lowered by replacing alarms with lights or silencing alarms quickly. 3. Dimmer switches should be used to shield the baby's eyes from bright lights with blankets over the top portion of the incubator. 5. Dimming the lights may encourage infants to open their eyes and be more responsive to their parents.

Maternal risks of occiput posterior (OP) malposition include which of the following? Note: Credit will be given only if all correct choices and no incorrect choices are selected. Select all that apply. 1. Blood loss greater than 1000 mL 2. Postpartum infection 3. Anal sphincter injury 4. Higher rates of vaginal birth 5. Instrument delivery

Answer: 2, 3, 5 Explanation: 2. Postpartum infection is a maternal risk of OP. 3. Anal sphincter injury is a maternal risk of OP. 5. Instrument delivery is a maternal risk of OP.

) Risk factors for labor dystocia include which of the following? Note: Credit will be given only if all correct choices and no incorrect choices are selected. Select all that apply. 1. Tall maternal height 2. Labor induction 3. Small-for-gestational-age (SGA) fetus 4. Malpresentation 5. Prolonged latent phase

Answer: 2, 4, 5 Explanation: 2. Labor induction is a risk factor of dystocia. 4. Malpresentation is a risk factor of dystocia. 5. Prolonged latent phase is a risk factor of dystocia.

Which of the following are considered risk factors for development of severe hyperbilirubinemia? Note: Credit will be given only if all correct choices and no incorrect choices are selected. Select all that apply. 1. Northern European descent 2. Previous sibling received phototherapy 3. Gestational age 27 to 30 weeks 4. Exclusive breastfeeding 5. Infection

Answer: 2, 4, 5 Explanation: 2. Previous sibling received phototherapy is considered a risk factor for development of severe hyperbilirubinemia. 4. Exclusive breastfeeding, particularly if nursing is not going well and excessive weight loss is experienced, is considered a risk factor for development of severe hyperbilirubinemia. 5. Infection is considered a risk factor for development of severe hyperbilirubinemia.

Benefits of skin-to-skin care as a developmental intervention include which of the following? Note: Credit will be given only if all correct choices and no incorrect choices are selected. Select all that apply. 1. Routine discharge 2. Stabilization of vital signs 3. Increased periods of awake-alert state 4. Decline in the episodes of apnea and bradycardia 5. Increased growth parameters

Answer: 2, 4, 5 Explanation: 2. Stabilization of vital signs is a benefit of skin-to-skin care as a developmental intervention. 4. Decline in the episodes of apnea and bradycardia is a benefit of skin-to-skin care as a developmental intervention. 5. Increased growth parameters are a benefit of skin-to-skin care as a developmental intervention.

When counseling a newly pregnant client at 8 weeks' gestation of twins, the nurse teaches the woman about the need for increased caloric intake. What would the nurse tell the woman that the minimum recommended intake should be? 1. 2500 kcal and 120 grams protein 2. 3000 kcal and 150 grams protein 3. 4000 kcal and 135 grams protein 4. 5000 kcal and 190 grams protein

Answer: 3 Explanation: 3. 4000 kcal and 135 grams protein is the recommended caloric and protein intake in a twin-gestation pregnancy.

A nurse explains to new parents that their newborn has developed respiratory distress syndrome (RDS). Which of the following signs and symptoms would not be characteristic of RDS? 1. Grunting respirations 2. Nasal flaring 3. Respiratory rate of 40 during sleep 4. Chest retractions

Answer: 3 Explanation: 3. A respiratory rate of 40 during sleep is normal.

A client is admitted to the labor and delivery unit with a history of ruptured membranes for 2 hours. This is her sixth delivery; she is 40 years old, and smells of alcohol and cigarettes. What is this client at risk for? 1. Gestational diabetes 2. Placenta previa 3. Abruptio placentae 4. Placenta accreta

Answer: 3 Explanation: 3. Abruptio placentae is more frequent in pregnancies complicated by smoking, premature rupture of membranes, multiple gestation, advanced maternal age, cocaine use, chorioamnionitis, and hypertension.

The client at 30 weeks' gestation is admitted with painless late vaginal bleeding. The nurse understands that expectant management includes which of the following? 1. Limiting vaginal exams to only one per 24-hour period. 2. Evaluating the fetal heart rate with an internal monitor. 3. Monitoring for blood loss, pain, and uterine contractibility. 4. Assessing blood pressure every 2 hours.

Answer: 3 Explanation: 3. Blood loss, pain, and uterine contractibility need to be assessed for client comfort and safety.

The nurse is caring for an infant who was delivered in a car on the way to the hospital and who has developed cold stress. Which finding requires immediate intervention? 1. Increased skin temperature and respirations 2. Blood glucose level of 45 3. Room-temperature IV running 4. Positioned under radiant warmer

Answer: 3 Explanation: 3. IV fluids should be warmed prior to administration and the newborn can be wrapped in a chemically activated warming mattress immediately following birth to decrease the postnatal fall in temperature that normally occurs.

A client in her second trimester is complaining of spotting. Causes for spotting in the second trimester are diagnosed primarily through the use of which of the following? 1. A nonstress test 2. A vibroacoustic stimulation test 3. An ultrasound 4. A contraction stress test

Answer: 3 Explanation: 3. Indirect diagnosis is made by localizing the placenta via tests that require no vaginal examination. The most commonly employed diagnostic test is the transabdominal ultrasound scan.

During discharge planning for a drug-dependent newborn, the nurse explains to the mother how to do which of the following? 1. Place the newborn in a prone position. 2. Limit feedings to three a day to decrease diarrhea. 3. Place the infant supine and operate a home apnea-monitoring system. 4. Wean the newborn off the pacifier.

Answer: 3 Explanation: 3. Infants with neonatal abstinence syndrome are at a significantly higher risk for sudden infant death syndrome (SIDS) when the mother used heroin, cocaine, or opiates. The infant should sleep in a supine position, and home apnea monitoring should be implemented.

The nurse is assessing a drug-dependent newborn. Which symptom would require further assessment by the nurse? 1. Occasional watery stools 2. Spitting up after feeding 3. Jitteriness and irritability 4. Nasal stuffiness

Answer: 3 Explanation: 3. Jitteriness and irritability can be an indicator of drug withdrawal.

The client vaginally delivers an infant that weighs 4750 g. Moderate shoulder dystocia occurred during the birth. During the initial assessment of this infant, what should the nurse look for? 1. Bell's palsy 2. Bradycardia 3. Erb palsy 4. Petechiae

Answer: 3 Explanation: 3. Macrosomic newborns should be inspected for cephalhematoma, Erb palsy, and fractured clavicles.

A laboring mother has recurrent late decelerations. At birth, the infant has a heart rate of 100, is not breathing, and is limp and bluish in color. What nursing action is best? 1. Begin chest compressions. 2. Begin direct tracheal suctioning. 3. Begin bag-and-mask ventilation. 4. Obtain a blood pressure reading.

Answer: 3 Explanation: 3. Most newborns can be effectively resuscitated by bag-and-mask ventilation.

The nurse assesses the gestational age of a newborn and informs the parents that the newborn is premature. Which of the following assessment findings is not congruent with prematurity? 1. Cry is weak and feeble 2. Clitoris and labia minora are prominent 3. Strong sucking reflex 4. Lanugo is plentiful

Answer: 3 Explanation: 3. Poor suck, gag, and swallow reflexes are characteristic of a preterm newborn.

If the physician indicates a shoulder dystocia during the delivery of a macrosomic fetus, how would the nurse assist? 1. Call a second physician to assist. 2. Prepare for an immediate cesarean delivery. 3. Assist the woman into McRoberts maneuver. 4. Utilize fundal pressure to push the fetus out.

Answer: 3 Explanation: 3. The McRoberts maneuver is thought to change the maternal pelvic angle and therefore reduce the force needed to extract the shoulders, thereby decreasing the incidence of brachial plexus stretching and clavicular fracture.

The nurse is caring for an infant born at 37 weeks that weighs 1750 g (3 pounds 10 ounces). The head circumference and length are in the 25th percentile. What statement would the nurse expect to find in the chart? 1. Preterm appropriate for gestational age, symmetrical IUGR 2. Term small for gestational age, symmetrical IUGR 3. Preterm small for gestational age, asymmetrical IUGR 4. Preterm appropriate for gestational age, asymmetrical IUGR

Answer: 3 Explanation: 3. The infant is preterm at 37 weeks. Because the weight is below the 10th percentile, the infant is small for gestational age. Head circumference and length between the 10th and 90th percentiles indicate asymmetrical IUGR.

What is the most significant cause of neonatal morbidity and mortality? 1. Amenorrhea 2. Posttraumatic stress disorder 3. Prematurity 4. Endometriosis

Answer: 3 Explanation: 3. The most significant cause of neonatal morbidity and mortality is prematurity and its associated complications such as respiratory distress syndrome, necrotizing enterocolitis, and intraventricular hemorrhage.

The parents of a preterm newborn wish to visit their baby in the NICU. A statement by the nurse that would not support the parents as they visit their newborn is which of the following? 1. "Your newborn likes to be touched." 2. "Stroking the newborn will help with stimulation." 3. "Visits must be scheduled between feedings." 4. "Your baby loves her pink blanket."

Answer: 3 Explanation: 3. The nurse always should encourage parents to visit and get to know their newborn, even in the NICU. Nurses foster the development of a safe, trusting environment by viewing the parents as essential caregivers, not as visitors or nuisances in the unit.

The nurse educator is describing the different kinds of abruptio placentae to a group of students, explaining that in a complete abruptio placentae, which of the following occurs? 1. Separation begins at the periphery of the placenta. 2. The placenta separates centrally and blood is trapped between the placenta and the uterine wall. 3. There is massive vaginal bleeding in the presence of almost total separation. 4. Blood passes between the fetal membranes and the uterine wall, and escapes vaginally.

Answer: 3 Explanation: 3. There is massive vaginal bleeding in the presence of almost total separation describes a complete separation of the placenta.

The nurse is caring for a client at 30 weeks' gestation who is experiencing preterm premature rupture of membranes (PPROM). Which statement indicates that the client needs additional teaching? 1. "If I were having a singleton pregnancy instead of twins, my membranes would probably not have ruptured." 2. "If I develop a urinary tract infection in my next pregnancy, I might rupture membranes early again." 3. "If I want to become pregnant again, I will have to plan on being on bed rest for the whole pregnancy." 4. "If I have aminocentesis, I might rupture the membranes again."

Answer: 3 Explanation: 3. There is no evidence that bed rest in a subsequent pregnancy decreases the risk for PPROM.

Lacerations of the cervix or vagina may be present when bright red vaginal bleeding persists in the presence of a well-contracted uterus. The incidence of lacerations is higher among which of the following childbearing women? Note: Credit will be given only if all correct choices and no incorrect choices are selected. Select all that apply. 1. Over the age of 35 2. Have not had epidural block 3. Have had an episiotomy 4. Have had a forceps-assisted or vacuum-assisted birth 5. Nulliparous

Answer: 3, 4 Explanation: 3. The incidence of lacerations is higher among childbearing women who undergo an episiotomy. 4. The incidence of lacerations is higher among childbearing women who undergo forceps-assisted or vacuum-assisted birth.

The nurse is working with parents who have just experienced the birth of their first child at 34 weeks. Which statements by the parents indicate that additional teaching is needed? Note: Credit will be given only if all correct choices and no incorrect choices are selected. Select all that apply. 1. "Our baby will be in an incubator to keep him warm." 2. "Breathing might be harder for our baby because he is early." 3. "The growth of our baby will be faster than if he were term." 4. "Tube feedings will be required because his stomach is small." 5. "Because he came early, he will not produce urine for 2 days."

Answer: 3, 4, 5 Explanation: 3. Preterm infants grow more slowly than do term infants because of difficulty in meeting high caloric and fluid needs for growth due to small gastric capacity. 4. Although tube feedings might be required, it would be because preterm babies have a marked danger of aspiration and its associated complications due to the infant's poorly developed gag reflex, incompetent esophageal cardiac sphincter, and inadequate suck/swallow/breathe reflex. 5. Although preterm babies have diminished kidney function due to incomplete development of the glomeruli, they can produce urine. Preterm infants usually have some urine output during the first 24 hours of life.

True postterm pregnancies are frequently associated with placental changes that cause a decrease in the uterine-placental-fetal circulation. Complications related to alternations in placenta functioning include which of the following? Note: Credit will be given only if all correct choices and no incorrect choices are selected. Select all that apply. 1. Increased fetal oxygenation 2. Increased placental blood supply 3. Reduced nutritional supply 4. Macrosomia 5. Risk of shoulder dystocia

Answer: 3, 4, 5 Explanation: 3. Reduced nutritional supply is a complication related to alternations in placenta functioning. 4. Macrosomia is a complication related to alternations in placenta functioning. 5. Risk of shoulder dystocia is a complication

Five clients are in active labor in the labor unit. Which women should the nurse monitor carefully for the potential of uterine rupture? Note: Credit will be given only if all correct choices and no incorrect choices are selected. Select all that apply. 1. Age 15, in active labor 2. Age 22, with eclampsia 3. Age 25, last delivery by cesarean section 4. Age 32, first baby died during labor 5. Age 27, last delivery 11 months ago

Answer: 3, 5 Explanation: 3. A woman who has had a previous cesarean section is at risk for uterine rupture. 5. A woman who does not have at least 18 months between deliveries is at greater risk for uterine rupture.

The nurse is assessing the newborn for symptoms of anemia. If the blood loss is acute, the baby may exhibit which of the following signs of shock? Note: Credit will be given only if all correct choices and no incorrect choices are selected. Select all that apply. 1. Increased pulse 2. High blood pressure 3. Tachycardia 4. Bradycardia 5. Capillary filling time greater than 3 seconds

Answer: 3, 5 Explanation: 3. Tachycardia would be a sign of shock. 5. Capillary filling time greater than 3 seconds would be a sign of shock.

The nurse admits into the labor area a client who is in preterm labor. What assessment finding would constitute a diagnosis of preterm labor? 1. Cervical effacement of 30% or more 2. Cervical change of 0.5 cm per hour 3. 2 contractions in 30 minutes 4. 8 contractions in 1 hour

Answer: 4 Explanation: 4. 8 contractions in a 60 minute period does define a diagnosis of preterm labor.

A 7 pound 14 ounce girl was born to an insulin-dependent type II diabetic mother 2 hours ago. The infant's blood sugar is 47 mg/dL. What is the best nursing action? 1. To recheck the blood sugar in 6 hours 2. To begin an IV of 10% dextrose 3. To feed the baby 1 ounce of formula 4. To document the findings in the chart

Answer: 4 Explanation: 4. A blood sugar level of 47 mg/dL is a normal finding; documentation is an appropriate action.

The client has delivered a 4200 g fetus. The physician performed a midline episiotomy, which extended into a third-degree laceration. The client asks the nurse where she tore. Which response is best? 1. "The episiotomy extended and tore through your rectal mucosa." 2. "The episiotomy extended and tore up near your vaginal mucous membrane." 3. "The episiotomy extended and tore into the muscle layer." 4. "The episiotomy extended and tore through your anal sphincter."

Answer: 4 Explanation: 4. A third degree laceration includes the anal sphincter.

The nurse caring for a postterm newborn would not perform what intervention? 1. Providing warmth 2. Frequently monitoring blood glucose 3. Observing respiratory status 4. Restricting breastfeeding

Answer: 4 Explanation: 4. Breastfeeding is an appropriate means of feeding for the postterm newborn.

The nurse is analyzing assessment findings on four newborns. Which finding might suggest a congenital heart defect? 1. Apical heart rate of 140 beats per minute 2. Respiratory rate of 40 3. Temperature of 36.5°C 4. Visible, blue discoloration of the skin

Answer: 4 Explanation: 4. Central cyanosis is defined as a visible, blue discoloration of the skin caused by decreased oxygen saturation levels and is a common manifestation of a cardiac defect.

The nurse is caring for a client in active labor. The membranes spontaneously rupture, with a large amount of clear amniotic fluid. Which nursing action is most important to undertake at this time? 1. Assess the odor of the amniotic fluid. 2. Perform Leopold maneuvers. 3. Obtain an order for pain medication. 4. Complete a sterile vaginal exam.

Answer: 4 Explanation: 4. Checking the cervix will determine whether the cord prolapsed when the membranes ruptured. The nurse would assess for prolapsed cord via vaginal examination.

A 38-week newborn is found to be small for gestational age (SGA). Which nursing intervention should be included in the care of this newborn? 1. Monitor for feeding difficulties. 2. Assess for facial paralysis. 3. Monitor for signs of hyperglycemia. 4. Maintain a warm environment.

Answer: 4 Explanation: 4. Hypothermia is a common complication in the SGA newborn; therefore, the newborn's environment must remain warm, to decrease heat loss.

A nurse is caring for a newborn on a ventilator who has respiratory distress syndrome (RDS). The nurse informs the parents that the newborn is improving. Which data support the nurse's assessment? 1. Decreased urine output 2. Pulmonary vascular resistance increases 3. Increased PCO2 4. Increased urination

Answer: 4 Explanation: 4. In babies with respiratory distress syndrome (RDS) who are on ventilators, increased urination/diuresis may be an early clue that the baby's condition is improving.

The nurse examines the client's placenta and finds that the umbilical cord is inserted at the placental margin. The client comments that the placenta and cord look different than they did for her first two births. The nurse should explain that this variation in placenta and cord is called what? 1. Placenta accreta 2. Circumvallate placenta 3. Succenturiate placenta 4. Battledore placenta

Answer: 4 Explanation: 4. In battledore placenta, the umbilical cord is inserted at or near the placental margin.

A newborn is receiving phototherapy. Which intervention by the nurse would be most important? 1. Measurement of head circumference 2. Encouraging the mother to stop breastfeeding 3. Stool blood testing 4. Assessment of hydration status

Answer: 4 Explanation: 4. Infants undergoing phototherapy treatment have increased water loss and loose stools as a result of bilirubin excretion. This increases their risk of dehydration.

The client is at 42 weeks' gestation. Which order should the nurse question? 1. Obtain biophysical profile today. 2. Begin nonstress test now. 3. Schedule labor induction for tomorrow. 4. Have the client return to the clinic in 1 week.

Answer: 4 Explanation: 4. Many practitioners use twice-weekly testing providing the amniotic fluid level is normal. One week is too long a period between assessments.

The client is carrying monochorionic-monoamniotic twins. The nurse teaches the client what this is, and the implications of this finding. The nurse knows that teaching is successful when the client states which of the following? 1. "My babies came from two eggs." 2. "About two thirds of twins have this amniotic sac formation." 3. "My use of a fertility drug led to this issue." 4. "My babies have a lower chance of surviving to term than fraternal twins do."

Answer: 4 Explanation: 4. Monochorionic-monoamniotic twins are both in one amniotic sac. There is an increased risk of umbilical cords becoming tangled or knotted and a higher incidence of fetal demise.

The nurse is caring for a newborn in the special care nursery. The infant has hydrocephalus, and is positioned in a prone position. The nurse is especially careful to cleanse all stool after bowel movements. This care is most appropriate for an infant born with which of the following? 1. Omphalocele 2. Gastroschisis 3. Diaphragmatic hernia 4. Myelomeningocele

Answer: 4 Explanation: 4. Myelomeningocele is a saclike cyst containing meninges, spinal cord, and nerve roots in thoracic and/or lumbar area. Meticulous cleaning of the buttocks and genitals helps prevent infection. The infant is positioned on abdomen or on side and restrain (to prevent pressure and trauma to sac). Hydrocephalus often is present.

The neonatal special care unit nurse is overseeing the care provided by a nurse new to the unit. Which action requires immediate intervention? 1. The new nurse holds the infant after giving a gavage feeding. 2. The new nurse provides skin-to-skin care. 3. The new nurse provides care when the baby is awake. 4. The new nurse gives the feeding with room-temperature formula.

Answer: 4 Explanation: 4. Preterm babies have little subcutaneous fat, and do not maintain their body temperature well. Formula should be warmed prior to feedings to help the baby maintain its temperature.

Intervention to reduce preterm birth can be divided into primary prevention and secondary prevention. What does secondary prevention include? 1. Diagnosis and treatment of infections 2. Cervical cerclage 3. Progesterone administration 4. Antibiotic treatment and tocolysis

Answer: 4 Explanation: 4. Secondary prevention strategies are antibiotic treatment and tocolysis.

Parents have been told their child has fetal alcohol syndrome (FAS). Which statement by a parent indicates that additional teaching is required? 1. "Our baby's heart murmur is from this syndrome." 2. "He might be a fussy baby because of this." 3. "His face looks like it does due to this problem." 4. "Cuddling and rocking will help him stay calm."

Answer: 4 Explanation: 4. The FASD baby is most comfortable in a quiet, minimally stimulating environment.

After delivery, it is determined that there is a placenta accreta. Which intervention should the nurse anticipate? 1. 2 L oxygen by mask 2. Intravenous antibiotics 3. Intravenous oxytocin 4. Hysterectomy

Answer: 4 Explanation: 4. The primary complication of placenta accreta is maternal hemorrhage and failure of the placenta to separate following birth of the infant. An abdominal hysterectomy may be the necessary treatment, depending on the amount and depth of involvement.

The nurse will be bringing the parents of a neonate with sepsis to the neonatal intensive care nursery for the first time. Which statement is best? 1. "I'll bring you to your baby and then leave so you can have some privacy." 2. "Your baby is on a ventilator with 50% oxygen, and has an umbilical line." 3. "I am so sorry this has all happened. I know how stressful this can be." 4. "Your baby is working hard to breathe and lying quite still, and has an IV."

Answer: 4 Explanation: 4. This answer is best because it explains what the parents will see in terminology that they will understand. A trusting relationship is essential for collaborative efforts in caring for the infant. The nurse should respond therapeutically to relate to the parents on a one-to-one basis.

The multiparous client at term has arrived to the labor and delivery unit in active labor with intact membranes. Leopold maneuvers indicate the fetus is in a transverse lie with a shoulder presentation. Which physician order is most important? 1. Artificially rupture membranes. 2. Apply internal fetal scalp electrode. 3. Monitor maternal blood pressure every 15 minutes. 4. Alert surgical team of urgent cesarean.

Answer: 4 Explanation: 4. This is the highest priority because vaginal birth is impossible with a transverse lie. Labor should not be allowed to continue, and a cesarean birth is done quickly.

Which nursing intervention is appropriate in the management of the preterm infant with hypothermia? Note: Credit will be given only if all correct choices and no incorrect choices are selected. Select all that apply. 1. Warm the baby rapidly to reverse the hypothermia. 2. Monitor skin temperature every 2 hours to determine whether the infant's temperature is increasing. 3. Keep IV fluids at room temperature. 4. Initiate efforts to maintain the newborn in a neutral thermal environment. 5. Warm the baby slowly to reverse hypothermia and reach a neutral thermal environment

Answer: 4, 5 Explanation: 4. The nurse should initiate efforts to block heat loss by evaporation, radiation, convection, and conduction. 5. The infant should be warmed slowly to prevent hypotension and apnea.

On assessment, a labor client is noted to have cardiovascular and respiratory collapse and is unresponsive. What should the nurse suspect? 1. An amniotic fluid embolus 2. Placental abruption 3. Placenta accreta 4. Retained placenta

answer 1 Explanation: 1. Cardiovascular and respiratory collapse are symptoms of an amniotic fluid embolus and cor pulmonale.


Kaugnay na mga set ng pag-aaral

Management 494 | Exam 2 Study Guide (Chapters 4-7)

View Set

Windows Activation 2.3.5 Practice Questions

View Set

Producing New and Digital Media Terms «M, N, & O»

View Set